App Math II For Engineering

You might also like

Download as pdf or txt
Download as pdf or txt
You are on page 1of 85

Applied Mathematics II

By:-Brhane Aregawi(MSc)
Mathematics Department
Mekelle University

October 29, 2020


Contents

1 Sequence and Series 2


1.1 Sequences . . . . . . . . . . . . . . . . . . . . . . . . . . . . . . . 2
1.1.1 Definition of Sequence . . . . . . . . . . . . . . . . . . . . 2
1.1.2 Types of Sequences . . . . . . . . . . . . . . . . . . . . . . 3
1.2 Convergence properties of Sequences . . . . . . . . . . . . . . . . 4
1.3 Infinite series . . . . . . . . . . . . . . . . . . . . . . . . . . . . . . 6
1.3.1 Convergence and Divergence properties of Infinite Series 6
1.4 Convergence and Divergence Tests . . . . . . . . . . . . . . . . . 7
1.4.1 Divergence Test . . . . . . . . . . . . . . . . . . . . . . . . 7
1.4.2 Test of convergence of a series . . . . . . . . . . . . . . . . 9
1.5 Alternating Series and Alternating Series Test . . . . . . . . . . . 12
1.5.1 Alternating Series Test . . . . . . . . . . . . . . . . . . . . 12
1.5.2 Absolute and Conditional Convergence . . . . . . . . . . 12
1.5.3 Ratio Test . . . . . . . . . . . . . . . . . . . . . . . . . . . . 13
1.5.4 Root Test . . . . . . . . . . . . . . . . . . . . . . . . . . . . 13
1.6 Generalized convergence tests . . . . . . . . . . . . . . . . . . . . 14

2 Power Series 15
2.1 Definition of Power Series at Any Point a . . . . . . . . . . . . . . 15
2.2 Convergence and Divergence, Radius and Interval of Convergence 16
2.3 Representations of Functions as Power Series . . . . . . . . . . . 16
2.4 Differentiation and Integration of Power Series . . . . . . . . . . 17
2.5 Taylor Series; Taylor Polynomial and Application . . . . . . . . . 18

3 Differential Calculus of Functions of Several Variables 20


3.1 Notations, Examples, Level Curves and Graphs . . . . . . . . . . 20
3.1.1 Notation and Examples . . . . . . . . . . . . . . . . . . . . 20
3.1.2 Graphs . . . . . . . . . . . . . . . . . . . . . . . . . . . . . 21
3.1.3 Level Curves . . . . . . . . . . . . . . . . . . . . . . . . . . 22
3.2 Limit and Continuity . . . . . . . . . . . . . . . . . . . . . . . . . 24
3.2.1 Limit . . . . . . . . . . . . . . . . . . . . . . . . . . . . . . 24
3.2.2 Continuity . . . . . . . . . . . . . . . . . . . . . . . . . . . 29
3.3 Partial Derivatives; Higher Order Partial Derivatives . . . . . . . 31
3.3.1 Partial Derivatives . . . . . . . . . . . . . . . . . . . . . . . 31
3.3.2 Higher Order Derivative . . . . . . . . . . . . . . . . . . . 33
3.4 Tangent Line . . . . . . . . . . . . . . . . . . . . . . . . . . . . . . 34
3.5 Directional Derivatives and Gradients . . . . . . . . . . . . . . . 35
3.5.1 Directional Derivatives . . . . . . . . . . . . . . . . . . . . 35
CONTENTS CONTENTS

3.5.2 Gradients . . . . . . . . . . . . . . . . . . . . . . . . . . . . 36
3.6 Tangent Plane and Tangent Plane Approximation . . . . . . . . . 38
3.6.1 Total Differential . . . . . . . . . . . . . . . . . . . . . . . 38
3.6.2 Tangent Plane . . . . . . . . . . . . . . . . . . . . . . . . . 38
3.6.3 Tangent Plane Approximation . . . . . . . . . . . . . . . . 40
3.7 Chain Rule and Implicitly Differentiation . . . . . . . . . . . . . 41
3.7.1 Chain Rule . . . . . . . . . . . . . . . . . . . . . . . . . . . 41
3.7.2 Implicitly Differentiation . . . . . . . . . . . . . . . . . . . 42
3.8 Relative Extrema of Function of Two Variables . . . . . . . . . . . 42
3.9 Largest and Smallest Values of a Function on a Given Set . . . . 45
3.10 Extreme Value under Constraint Condition . . . . . . . . . . . . 46
3.10.1 Method of Lagrange Multipliers for One Constraint . . . 46
3.10.2 Method of Lagrange Multipliers for Two Constraints . . . 49

4 Multiple Integral 53
4.1 Double Integrals and Their Evaluation by Iterated Integrals . . . 53
4.1.1 Definition of Double Integral . . . . . . . . . . . . . . . . 53
4.1.2 Iterated Integral . . . . . . . . . . . . . . . . . . . . . . . . 54
4.1.3 Properties of double integrals . . . . . . . . . . . . . . . . 57
4.1.4 Double Integral over vertically and horizontally simple
regions . . . . . . . . . . . . . . . . . . . . . . . . . . . . . 57
4.1.5 Change of Variables in Double Integral . . . . . . . . . . . 61
4.1.6 Double Integral in Polar Coordinate . . . . . . . . . . . . 63
4.1.7 Application of Double Integral: Area, volume, Center of
Mass of Plane Region, Surface Area . . . . . . . . . . . . . 65
4.2 Triple Integral . . . . . . . . . . . . . . . . . . . . . . . . . . . . . 72
4.2.1 Notation of Triple Integral . . . . . . . . . . . . . . . . . . 72
4.2.2 Triple Integral in Cylindrical Coordinate . . . . . . . . . . 74
4.2.3 Triple Integral in Spherical Coordinate . . . . . . . . . . . 75
4.2.4 Application of Triple Integral: Volume, Center of Mass
of Solid Region . . . . . . . . . . . . . . . . . . . . . . . . 77

1
Chapter 1

Sequence and Series

1.1 Sequences
1.1.1 Definition of Sequence
A sequence is a list of numbers written in a specific order. A Sequence is a set
can be defined as a function whose domain is the set of positive integers. A
sequence can be written in a definite order:

a1 , a 2 , a 3 , · · · , a n , · · ·

Each ai are called the terms of the sequence. The numbers


a1 is the first term
a2 is the second term
..
.
an is the nth term.
Notice that for every positive integer n there is a corresponding number an and
so a Sequence can be defined: an = f (n), n ∈ Z.
Notation: The sequence {a1 , a2 , a3 , · · · , an , · · · } is also denoted by {an } or {an }∞
n=i ,
where i = 0, 1, 2, 3, · · · is called the index of the sequence.
A Sequence is like a set, except:

1. the terms are in order (with Sets the order does not matter)
2. the same value can appear many times (only once in Sets)

Example 1 1. {0, 1, 0, 1, 0, 1...} is the sequence of alternating 0s and 1s.

2. the sequence {3, 5, 7, 9...} starts at 3 and jumps 2 every time; then find
a. the nth term or the general formula of the sequence,
b. the 10th and 100th terms.
( n )
−1
3. Calculate the first four terms of the sequence: {an} = .
n n=1

2
1 Applied Math. II

Exercise 1 Calculate the first five terms of the following sequences


)∞
(−1)n (n + 1)
( ( !)
n ∞ nΠ
 
a) b) c) cos
n + 1 n=1 3n n=0 6 n=0

1.1.2 Types of Sequences


A) Infinite or Finite sequence: based on the numbers of the terms of the se-
quence. If the sequence goes on forever it is called an infinite sequence,
otherwise it is a finite sequence.

Example 2 Examples of finite sequences:


a) {1, 3, 5, 7} b){4, 3, 2, 1, · · · , -50}
c) {a, b, c, d, e} d) {f, r, e, d}

Example 3 Examples of infinite sequences:


a) {0, 1, 0, 1, 0, 1 ...} is the sequence of alternating 0s and 1s (yes they are in
order, it is an alternating order in this case).
b) {1, 2, 3, 4...}.
c) {20, 25, 30, 35 ...}.
d) {1, 2, 4, 8, 16, 32 ...}.

B) Increase and decrease sequences.


C) bounded and unbounded sequences.
D) Converge and diverge sequences.
E) Arithmetic Sequences: with common difference between one term and the
next.

Example 4 Consider the sequence 1, 4, 7, 10, 13, 16, 19, 22, 25, · · · :common dif-
ference, d=3 and then its nth term is An = 3n − 2.

⇒ In General we could write an arithmetic sequence {An } as:

{A1 , A1 + d, A1 + 2d, A1 + 3d, · · · }

where; a is the first term, and d is the difference between the terms (Usu-
ally called the "common difference") so that its nth term is

An = A1 + (n − 1)d.

F) Geometric Sequences: In a geometric sequence each term is found by mul-


tiplying the previous term by a constant.
Example: Consider the sequence 2, 4, 8, 16, 32, 64, 128, 256, · · · . This se-
quence has a factor of 2 between each number. Then its Rule is

an = 2 n

By Brhane Aregawi Math Department, MU


3
1 Applied Math. II

. ⇒ In General we could write an geometric sequence {Gn } as:

{G1 , G1 r, G1 r 2 , G1 r 3 , ...}

Where: G1 is the first term, and r is the factor between the terms (called
the "common ratio").
Note that:
i. r , 0.
ii. The nth term is Gn = G1 r n−1 .

1.2 Convergence properties of Sequences


Definition 1 Let {an }∞ ∞
n=m be a sequence. A number L is the limit of {an }n=m if for
every  there is an integer N such that if n > N , then |an − L| < . In this case we
write, limn→∞ an = L if such a number L exists, we say that {an }∞n=m converges to L
or that limn→∞ an = L exists. If such a number L does not exist, we say that {an }∞
n=m
diverges or that limn→∞ an = L does not exist.
Note that: as limit of functions, the limit of a sequence is unique if it exists.

Example 5 Using limit evaluetion determine the convergence or divergence of the


following sequences.
 )∞
1 n
(
n
1. an = n+1 2. 1 +
n n=1
 n
2n 1
3. {(−4) } 4. { 3 }
2
5. { 6n4n+1
2 } 6. {1, 2, 3, 21 , 14 , 16 , · · · , 2n
1
,···}
7. Show that {r n } is converges only for −1 < r < 1.
(
n n 1, if n is even;
8 {(−1) } : limn→∞ (−1) =
−1, if n is odd.

Rules of Convergence sequences: Let {an }∞ ∞


n=m and {bn }n=m be converge to L
and K respectively. Then, for any constant c:

a1 . {an ± bn } be converges and lim (an ± bn ) = L ± K


n→∞

a2 . {can } is converges and lim (can ) = cL.


n→∞

a3 . {an bn } is converges and lim (an bn ) = LK.


n→∞

an L
a4 . { ban } is converges and lim = , provided K , 0.
n n→∞ bn K
p p
a5 . {an } is converges and lim an = Lp .
n→∞

Theorem 1 Given the sequence {an } if we have a function f (x) such that f (n) = an
and lim f (x) = L then lim an = L.
x→∞ n→∞

By Brhane Aregawi Math Department, MU


4
1 Applied Math. II

Theorem 2 If lim |an | = 0 then lim an = 0. But the converse is not true.
n→∞ n→∞

Example 6 Determine the convergence of:


2
3n −1 ∞ 2n
a) { 10n+5n2 }n=1 b) { en }∞
n=1

(−1)n ∞ (−1)2n ∞
c) { n }n=1 d) { n }n=0

Remmember that; Limit Evaluetion methods:


* Limit rules ( Addition, constant mul., product, questient and power)
* Direct substitution
* Common factor cancellation

* Composition substitution
* Sqeezing (or sundwich)theorem
* L’hopitals rule

Definition 2 Given any sequence {an }. Then {an } is;

1. increasing seq. if an ≤ an+1 for every n.


2. decreasing seq. if an ≥ an+1 for every n.

3. an increasing or is a decreasing sequence we call it is monotonic sequence.


4. bounded below if there exists a number m such that m ≤ an for every n. The
number m is sometimes called a lower bound for the sequence.
5. bounded above if there exists a number M such that M ≥ an for every n. The
number M is sometimes called a upper bound for the sequence.
6. If the sequence is both bounded below and bounded above we call the sequence is
bounded.

Theorem 3 If {an } is bounded and monotonic then {an } is convergent.

Example 7 Determine if the sequence is monotonic and/or bounded.

i. {(−n2 )}∞
n=0 ii. {(−1)n+1 }∞
n=1 iii. { n12 }∞
n=5

n ∞ n ∞ 3
n
iv. { n+2 }n=0 v. { n+1 }n=1 vi. { n4 +10000 }∞
n=0

Note(Application of derivative): any function f(x) is increasing if f 0 (x) ≥ 0 ∀x


and decreasing if f 0 (x) ≤ 0 ∀x in its domain.

By Brhane Aregawi Math Department, MU


5
1 Applied Math. II

1.3 Infinite series


Definition: A series is simply the sum of terms of the sequence and the infinite
series is given as:

X X
a1 + a2 + a3 + · · · + an + · · · = an = an .
n=1
P
n is called index of the summation( or simply index) and the symbol is called
series notation or summation notation or sigma notation of the sequnce.


X
Example 8 i. 2 + 4 + 6 + 8 + 10 + · · · + · · · = 2n.
n=1

n(n+1)
ii. 1 + 2 + 3 + 4 + + · · · + n = 2 .

X 1 1 1 1 1 1
iii. n
= + + + + · · · + n + · · · = 1.
2 2 4 8 16 2
n=1

1.3.1 Convergence and Divergence properties of Infinite Se-


ries

X X
Consider the infinite series an = an
n=1
⇒ finite summations/series of the given series are:
S1 = a1
S2 = a1 + a2
S3 = a1 + a2 + a3
S 4 = a1 + a2 + a3 + a4
S5 = a1 + a2 + a3 + a4 + a5
..
.
n
X
Sn = a1 + a2 + a3 + a4 + a5 + · · · + an = ai
i=1

These are called partial sums. Notice that the partial sums will form an infinite
sequence, {sn }∞
n=1 .

Definition 3 The sequence an is convergent if its partial sums sequence {sn }∞


P
n=1
is convergent, that is if lim sn = s exists (as a finite number), we call the sum of the
n→∞
X∞
P P
infinite series an and an = s, otherwise an is a divergent series.
n=1

Example 9 Determine if the following series is convergent or divergent. If it con-


verges determine its sum.

By Brhane Aregawi Math Department, MU


6
1 Applied Math. II

∞ ∞
X X 1 3 1 1
a. n b. : sn = 4 − 2n − 2(n+1)
n2 − 1
n=1 n=2
∞ ∞
X
n
X 1
c. (−1) d. is a divergent series
n
n=0 n=1
P
Theorem 4 If an converges then lim an = 0. But the converse is not true.
n→∞

∞ ∞
1 1 X 1 X 1
Example 10 lim = 0 and lim 2 = 0, but is divergent and is
n→∞ n n→∞ n n n2
n=1 n=1
convergent [we see later].

1.4 Convergence and Divergence Tests


1.4.1 Divergence Test
X
Theorem 5 a) If lim an , 0 then an diverges.
n→∞

X
b) If an converges, then lim an = 0. But the converse is not true.
n→∞


X 1 1
Example 11 i. is diverges, but lim = 0.
n n→∞ n
n=1

X 1 1
ii. 2
is converges and lim 2 = 0.
n n→∞ n
n=1

X 4n2 − n3 4n2 − n3 −1
iii. Using Divergence Test; is diverges, since lim = , 0.
10 + 2n3 n→∞ 10 + 2n3 2
n=0

F Geometric Series

X
Theorem 6 The Geometric series ar n−1 = a + ar + ar 2 + ar 3 + · · · + ar n−1 + · · · is
n=0

X a
convergent if |r| < 1 and its sum is ar n−1 = , if, |r| > 1 the geometric series
1−r
n=0
is divergent. Here r is called the common ratio.

X
Example 12 Is the series 22n 31−n convergent or divergent?
n=1

F Telescoping Series
This name comes from what happens with the partial sums of the sequence
such that we must get terms to cancel.

By Brhane Aregawi Math Department, MU


7
1 Applied Math. II


X 1
Example 13 Show that the series is convergent, and find its sum.
n(n + 1)
n=1

Exercise 2 Determine if the following series converges or diverges. If it converges


find its value.

X 1
i.
n2 + 3n + 2
n=0

X 1
ii.
n2 + 4n + 3
n=1
∞ ∞ 
3 + 2n 1 1
X X 
iii. = + is not a telescoping series.
n2 + 3n + 2 n+1 n+2
n=1 n=1

X 1
F Harmonic Series The Harmonic Series is which is divergent.
n
n=1


X 1
Example 14 i. Show that the series 5 is divergent.
n
n=1

X 1
ii. Show that the series is divergent
n
n=4
The convergence of the series is not change when:
adding/subtracting a finite number (constant) from a series.

multiplying a series by a constant.


F Combination of Series

X ∞
X ∞
X
Let an and bn be two give series. Then the two new series, (an +
n=1 n=1 n=1

X
bn ) and can whose convergence is generated by the convergence of original
n=1
series.

X ∞
X ∞
X
Theorem 7 1. If an and bn be converge, then (an + bn ) converges and
n=1 n=1 n=1

X ∞
X ∞
X
(an + bn ) = an + bn .
n=1 n=1 n=1

X ∞
X ∞
X ∞
X
2. an is converges, then can also converges and can = c an
n=1 n=1 n=1 n=1

∞ !
X 4 2
Example 15 Show that the series − converges, and find its sum.
2n n(n + 1)
n=1

By Brhane Aregawi Math Department, MU


8
1 Applied Math. II

1.4.2 Test of convergence of a series


Determine whether a series is convergent or divergent without explicitly find-
ing its sum.
A. Integral Test
This method also enable us to find good estimates of the sum.

X 1 1
♠ Show that the harmonic series is divergent series. Let f (x) = x on
n
n=1
the interval [1, ∞), the improper integral
Z∞
1
1 x

is called integral divergent. Also to prove that the harmonic series di-
verges using estimate the area by breaking up the interval into segments
and then sketching in rectangles and using the sum of the area all of the
rectangles as an estimate of the actual area. Break up the interval into
sub-intervals of width 1 and we will take the function value at the left
endpoint as the height of the rectangle.

So, the
 area under
  the curve
 is approximately,
  
A ≈ 1 (1) + 2 (1) + 3 (1) + 14 (1) + 51 (1) + · · ·
1 1 1
         
= 11 + 12 + 13 + 14 + 15 + · · · .
First, each of the rectangles over estimates the actual area and secondly
the formula for the area is exactly the harmonic series!
∞ Z∞
X 1 1
A≈ > =∞
n 1 x
n=1

X 1
This implies = ∞.
n
n=1
that is the improper integral and the series have exactly the same conver-
gence.
We know that by divergence test the series

X 1
n2
n=1

converges. Similarly to the above process to prove this: let f (x) = x12 is on
Z∞
1
the interval [1, ∞). Then the Improper Integral 2
dx = 1 and so this
1 x

By Brhane Aregawi Math Department, MU


9
1 Applied Math. II

integral converges.
Again try to estimate the area under this curve using the right end points
for the height of our rectangles. Here is a sketch of this case,

In this case the area estimation is,


1 1 1 1 1
         
A ≈ (1) + (1) + (1) + (1) + (1) + · · ·
22 32 42 52 62
1 1 1 1 1
         
= + 2 + 2 + 2 + 2 + ···
22 3 4 5 6
 2  2  2  2  2
1 1 1 1 1
= + + + + + ···.
2 3 4 5 6
Now, this implies that
∞  2  2  2  2  2  2
X 1 1 1 1 1 1 1
= + + + + + + ···
n2 1 2 3 4 5 6
n=1
Z∞
1
< 1+ 2
dx = 1 + 1 = 2.
1 x

X 1
Put this in together, we get; <2
n2
n=1
Integral Test: Suppose that f (x) is a positive, decreasing function on the
interval [k, ∞) and that an = f (n) then,
Z∞ X ∞
i. if f (x) is convergent so is an .
n=k n=k
Z ∞ ∞
X
ii. if f (x) is divergent so is an .
n=k n=k
This test does not give the value of a series. It will only give the
convergence/divergence of the series.

Example 16 Using integral test determine if the following series is con-


vergent or divergent.
∞ ∞ ∞
X 1 X 1 X 2
a. b. c. ne−n
n n ln(n) n=o
1 n=2
∞ ∞
X 1 X 1
d. e.
n2 + 1 n2
1 1

By Brhane Aregawi Math Department, MU


10
1 Applied Math. II

B. P-Series Test

X 1
If k > 0 then converges if p>1 and diverges if p ≤ 1. This series in
np
n=k
this fact are called p-series and so this fact is called the p-series test.

Example 17 Determine if the following series is convergent or divergent.


∞ ∞ ∞
X 1 X 1 X 1
a. b. √ c.
n7 n n3
1
1 n=2 1

The Integral Test applicable if the series terms MUST be positive. If they
are negative then the test doesn’t work.
C. The comparison Test X X
Suppose that we have two series an and bn with, 0 ≤ an ≤ bn for all
n. Then,
X X
1. If bn is convergent so is an .
X X
2. If an is divergent so is bn .
X
3. bn is divergent, then we haven’t conclusion about the convergence
X
of an .

In other words, we have two series of positive terms and the terms of one
of the series is always larger than the terms of the other series. Then if
the larger series is convergent the smaller series must also be convergent.
Likewise, if the smaller series is divergent then the larger series must also
be divergent.
In the comparison tests the idea is to compare a given series with a series
that is known to be convergent or divergent

Example 18 Determine if the series is convergent or divergent.


∞ ∞ ∞
X 1 X n X n2 + 1
a. b. c.
2n + 1 n2 − cos2 n n4 + 5
n=1 n=1 n=1
∞ ∞
X 5 X 1
d. e. (?)
2n2 + 4n + 3 3n − n
n=1 n=0

D. Limit Comparison Test X X


Suppose that we have two series an and bn with, an , bn ≥ 0 for all
n. Define,
a
c = lim n
n→∞ bn
If c is positive (i.e. c > 0 ) and is finite (i.e. c < ∞ ) then either both series
converge or both series diverge.

Example 19 Determine if the series is convergent or divergent.


∞ ∞ ∞
X 1 X 4n2 + n X 4n − 3
a. b. √ c. .
3n − n 3 7
n +n 3 n3 − 5n − 7
n=0 n=2 n=1

By Brhane Aregawi Math Department, MU


11
1 Applied Math. II

1.5 Alternating Series and Alternating Series Test


Definition 4 An alternating series is a series whose terms are alternately positive
and negative. The nth term of an alternating series is of the form

an = (−1)n−1 bn or an = (−1)n bn

Where each bn is a positive number (In fact, bn = |an |).

1.5.1 Alternating Series Test


X
Suppose that we have a series an and either an = (−1)n bn or an = (−1)n+1 bn
where bn ≥ 0 for all n. Then if,
i. lim bn = 0 and,
n→∞
X
ii. {bn } is eventually a decreasing sequence then, the series an is convergent.

Example 20 Determine if the following series is convergent or divergent.


∞ ∞
X (−1)n+1 X (−1)n n2
a. b.
n n2 + 5
n=1 n=1
∞ √ ∞
X (−1)n−3 n X (cosnπ
c. d. √ .
n+4 n
n=0 n=1

1.5.2 Absolute and Conditional Convergence


X
Definition 5 A series an is called absolutely convergent if the series of absolute
X
values |an | is convergent.
X
A series an is called conditionally convergent if it is convergent but not abso-
lutely convergent.
Notice that
X
If an is a series with positive terms, then |an | = an and so absolute convergence
is the same as convergence in this case.
X
If an is absolutely convergent, then it is also convergent.

Example 21 a. Show that alternating harmonic series is conditionaly convergent.



X 1
b. Show that that is absolutely convergent.
n2
n=1

By Brhane Aregawi Math Department, MU


12
1 Applied Math. II

1.5.3 Ratio Test


Definition 6 (Factorial definition) If n is an integer such that n ≥ 0, then n fac-
torial is defined as,
n! = n(n − 1)(n − 2)(n − 3) · · · 4.3.2.1 if n ≥ 1 and 0! = 1! = 1.
Note that: (2n)! , 2n!.
X an+1
Ratio Test: Suppose we have the series an . Define, r = lim then,
n→∞ an

1. If r < 1 the series is absolutely convergent (and hence convergent).


2. If r > 1 the series is divergent.
3. If r =1 the series may be divergent, convergent, or absolutely convergent i.
e we cannot draw any conclusion about the convergence.
Notice that; if r = 1 the ratio test is pretty much worthless and we would need
to resort to a different test to determine the convergence of the series.
This test will be particularly useful for series that contain factorials.

Example 22 Determine if the series is convergent or divergent.


∞ ∞
X n3 X n!
1. (−1)n n 2. .
3 5n
n=1 n=0
∞ ∞
X (−1)n X (−10)n
3. 4. .
n2 + 1 42n+1 (n + 1)
n=0 n=1

1.5.4 Root Test


This test is convenient to apply when powers occur.
1
X p
Root Test: Suppose we have the series an . Define, r = lim n |an | = lim |an | n
n→∞ n→∞
then,
1. If r < 1 the series is absolutely convergent (and hence convergent).
2. If r > 1 the series is divergent.
3. If r =1 the series may be divergent, convergent, or absolutely convergent i.
e we cannot draw any conclusion about the convergence.
Notice that; if r = 1 the ratio test is pretty much worthless and we would need
to resort to a different test to determine the convergence of the series.
1
As an exercise for the reader please show that lim n n = 1.
n→∞

Example 23 Determine if the series is convergent or divergent.


∞ ∞ 
nn 2n + 3
X X 
1. 2.
31+2n 3n + 2
n=1 n=1
∞ ∞
X 2n X 2n
3. 4.
n n2
n=1 n=1

As follow we obtain convergence tests that apply to any series, non-negative or


not.

By Brhane Aregawi Math Department, MU


13
1 Applied Math. II

1.6 Generalized convergence tests


Theorem 8 (Generalized convergence tests)

X
Let an be a series.
n=1

X
a. Generalized comparison test: If |an | ≤ |bn | for n ≥ 1, and if |bn | converges
n=1

X
then an converges (absolutely).
n=1

a
b. Generalized Limit comparison test: If L = lim n , where L is a positive
n→∞ bn

X ∞
X
number, and if |bn | converges then an converges (absolutely).
n=1 n=1

a
c. Generalized ratio test: Suppose that an , 0 and that lim n+1 = r(possibly ∞).
n→∞ an

X X∞
If r < 1 then, an converges (absolutely). If r > 1 then an diverges. If
n=1 n=1
r = 1, then from this test alone we cannot draw any conclusion about the
convergence of the series.
p
d. Generalized root test: Suppose that lim n |an | = r(possibly ∞). If r < 1 then,
n→∞
X∞ ∞
X
an converges (absolutely). If r > 1 then an diverges. If r = 1, then from
n=1 n=1
this test alone we cannot draw any conclusion about the convergence of the
series.
∞ n
X x
Exercise 3 a. Show that converges absolutely for |x| < 1, converges condi-
n
n=1
tionally for x = −1, and diverges for |x| > 1 and for x = 1.

X n!
b. Determine weather the series (−1)n+1 n converges or diverges.
10
n=1

By Brhane Aregawi Math Department, MU


14
Chapter 2

Power Series

2.1 Definition of Power Series at Any Point a


Definition 7 A power series is a series of the form

X
cn xn = c0 + c1 x + c2 x2 + c3 x3 + · · · (2.1)
n=0

where x is a variable and the cn0 s are constants called the coefficients of the series.
For each fixed x, 2.1 the series is a series of constants that we can test for con-
vergence or divergence. A power series may converge for some values of x and
diverge for other values of x. The sum of the series is a function

f (x) = c0 + c1 x + c2 x2 + c3 x3 + · · ·

whose domain is the set of all for which the series converges. Notice that f
resembles a polynomial. The only difference is that has infinitely many terms.
For instance, if we take cn = 1 for all n, the power series becomes the geometric
series

X
xn = 1 + x + x2 + x3 + · · ·
n=0
which converges when |x| < 1 and diverges when |x| ≥ 1.
More generally, a series of the form

X
cn (x − a)n = c0 + c1 (x − a) + c2 (x − a)2 + c3 (x − a)3 + · · · (2.2)
n=0

is called a power series in (x − a) or a power series centered at a or a power


series about a. Notice that in writing out the term corresponding to n = 0 in
2.1 and 2.2 we have adopted the convention that (x − a)0 = 1 even when x = a.
Notice also that when x = a all of the terms are 0 for n ≥ 1 and so the power
series 2.2 always converges when x = a.

X
Example 24 For what values of x is the series n!xn convergent?
n=0

15
2 Applied Math. II

2.2 Convergence and Divergence, Radius and Inter-


val of Convergence

X
Theorem 9 For a given power series cn (x − a)n there are only three possibilities:
n=0

(i) The series converges only when x = a.[R = 0, x = a]


(ii) The series converges for all x.[R = ∞, I = (∞, ∞)]
(iii) There is a positive number R such that the series converges if |x − a| < R and
diverges if |x − a| > R.

The number R in three cases above is called the radius of convergence of the
power series. The interval of convergence of a power series is the interval that
consists of all values of x for which the series converges.

Series Radius of convergence Interval of convergence


X∞
xn R=1 (−1, 1)
n=0

X
n!xn R=0 {0}
n=0

X (x − 3)n
R=1 [2, 4)
n
n=1

X (−1)n x2n
R=∞ (∞, ∞)
22n (n!)2
n=0

In general, the Ratio Test (or sometimes the Root Test) should be used to deter-
mine the radius of convergence . The Ratio and Root Tests always fail when is
an endpoint of the interval of convergence, so the endpoints must be checked
with some other test.

Example 25 Find the radius of convergence and interval of convergence of the se-
ries

X (−3)n xn

n=0 n+1
Answer: R = 3 and I = (−5, 1)

2.3 Representations of Functions as Power Series


Represent certain types of known functions as sums of power series (or in-
finitely many terms) using geometric series or by differentiating or integrating
such a series.
This strategy is useful for integrating functions that don’t have elementary
anti-derivatives, for solving differential equations, and for approximating func-
tions by polynomials.

By Brhane Aregawi Math Department, MU


16
2 Applied Math. II

1
The power series representation of f (x) = 1−x is given as

1 X
= 1 + x + x2 + x3 + · · · = xn , |x| < 1
1−x
n=0

which is a geometric series with a = 1 and r = x.


1
Example 26 Express as the sum of a power series and find the interval of
1+x2

X
convergence. [Ans. f (x) = (−1)n x2n ]
n=0


X (−1)n n
Example 27 Find a power series representation for 1
x+2 . [Ans. f (x) = 2n+1 ]
x
n=0


x3
X (−1)n n+1
Example 28 Find a power series representation for x+2 .[Ans. f (x) = 2n+1 ]
x
n=0

2.4 Differentiation and Integration of Power Series


X
Theorem 10 If the power series cn (x −a)n has radius of convergence R > 0, then
the function f defined by

X
f (x) = c0 + c1 (x − a) + c2 (x − a)2 + c3 (x − a)3 + · · · = cn (x − a)n
n=0

is differentiable (and therefore continuous) on the interval (a − R, a + R) and



X ∞
X
(i) f 0 (x) = c1 +2c2 (x−a)+3c3 (x−a)2 +· · · = ncn (x−a)n−1 = (n+1)cn+1 (x−
n=1 n=0
a)n

R (x−a)2 (x−a)3 (x−a)4 (x − a)n+1 X
(ii) f (x)dx = C+c0 (x−a)+c1 2 +c2 3 +c3 4 +· · · = C+ cn
n+1
n=0
The radii of convergence of the power series in Equations (i) and (ii) are both
R.
1
Example 29 Express (1−x) 2 as a power series by differentiating method and what is

the radius of convergence?



X ∞
X
Ans. f (x) = nxn−1 = (n + 1)xn , R = 1 and I = (−1, 1)
n=1 n=0

Example 30 Find a power series representation for ln(1 + x) and its radius of con-
vergence.

X (−1)n xn+1
Ans. f (x) = C + V C = 0, R = 1 and I = (−1, 1]
n+1
n=0

By Brhane Aregawi Math Department, MU


17
2 Applied Math. II

Example 31 Find a power series representation for f (x) = tan−1 x.



X (−1)n x2n+1
Ans. f (x) = C + V C = 0, R = 1 and I = [−1, 1]
2n + 1
n=0

Example 32 Using the above result show that:


∞ ∞
π
X (−1)n π
X (−1)n
(a) 4 = and 6 =
2n + 1
n=0 n=0
(2n + 1)3n+1/2
∞ ∞
X n+1 1
X (−1)n
(b) 4 = and 2 =
2n 2n+1
n=0 n=0
∞ ∞
X (−1)n X (−1)n
(c) ln 32 = and ln 2 =
(n + 1)2n+1 n+1
n=0 n=0
R
1
Example 33 (a) Evaluate 1+x7
dx as a power series.

X (−1)n x7n+1
Ans. f (x) = C +
7n + 1
n=0
R 0.5
1
(b) Use part (a) to approximate 0 1+x7
dx correct to within 10−7 .

2.5 Taylor Series; Taylor Polynomial and Applica-


tion
Theorem 11 If f has a power series representation (expansion) at a, that is, if

X
f (x) = cn (x − a)n , x − a < R
n=0

then its coefficients are given by the formula


f n (a)
cn =
n!
By substitution, we obtain

X f n (a)(x − a)n
f (x) =
n!
n=0
f 0 (a)(x − a) f 00 (a)(x − a)2 f 000 (a)(x − a)3 f (4) (a)(x − a)4
= f (a) + + + + + ···
1! 2! 3! 4!
This series is called the Taylor series of the function f at a(or about a or cen-
tered at a). For the special case a = 0 the Taylor series becomes
f 0 (a)x f 00 (a)x2 f 000 (a)x3 f (4) (a)x4
f (x) = f (a) + + + + + ···
1! 2! 3! 4!
This case arises frequently enough that it is given the special name Maclaurin
series.

By Brhane Aregawi Math Department, MU


18
2 Applied Math. II

Example 34 Find the Maclaurin series of the function f (x) = ex and its radius of
convergence.

As with any convergent series, this means that f (x) is the limit of the sequence
of partial sums. The partial sums of the Taylor series are:
n
X f i (a)(x − a)i
Tn (x) =
i!
i=0
f 0 (a)(x − a) f 00 (a)(x − a)2 f 000 (a)(x − a)3 f (n) (a)(x − a)n
= f (a) + + + + ··· +
1! 2! 3! n!

Notice that Tn is a polynomial of degree n called the nth -degree Taylor poly-
nomial of f at a.

Example 35 Find the Taylor polynomials at 0 (or Maclaurin polynomials) with


n = 1, 2, and 3 of f (x) = ex .

Theorem 12 If f (x) = Tn (x) + Rn (x), where Rn (x) is called the remainder of the
Taylor series of f at a and
lim Rn (x) = 0
n→∞

for |x−a| < R, then f is equal to the sum of its Taylor series on the interval |x−a| < R.

Theorem 13 (Taylor’s Inequality) If |f (n+1) (x)| ≤ M on

|x − a| ≤ d

, then the remainder Rn (x) of the Taylor series satisfies the inequality

M
|Rn (x)| ≤ (x − a)n+1 for|x − a| ≤ d
(n + 1)!

Example 36 Prove that ex is equal to the sum of its Maclaurin series.

Example 37 Find the Taylor series for ex at a = 2.

Example 38 Find the Maclaurin series for sin x and prove that it represents sin x
for all x.

Exercise 4 Find the Maclaurin series for each of the ff function:

(i) f (x) = cos x (ii) f (x) = x cos x (iii) f (x) = sin x at a = π/3

k !
X k n
The Muclarine series of f (x) = (1 + x)k is x , where k is any real number.
n
n=0
This series is called the binomial series for |x| < 1.

By Brhane Aregawi Math Department, MU


19
Chapter 3

Differential Calculus of
Functions of Several
Variables

3.1 Notations, Examples, Level Curves and Graphs


3.1.1 Notation and Examples
Definition 8 A function of several variables is a rule that assign a unique real
number f (x1 , x2 , · · · , xn ) to each ordered n-tuple of real numbers (x1 , x2 , · · · , xn ) in
the domain of the function; that is, f : D ⊆ Rn → R.

Example 39 f (x, y) = xy 2 and g(x, y) = x2 − y 2 are both examples of function of


two variables.

The domain of function of several variables is taken to be the set of all values
of the variables for which the function is defined.

Example 40 Find the domain of the following functions.

cos(x + z) 2x
(a) f (x, y) = x ln y (b) f (x, y, z) = (c) g(x, y) =
xy y − x2
q q q
(d) f (x, y) = ln 4 − x2 − y 2 (e) f (x, y, z) = ln x2 + y 2 − 4 (f ) g(x, y) = 9 − x2 − y 2 − z2

Properties: The sum, product, and quotient of two functions f and g of several
variables are defined as
1. (f ± g)(x, y) = f (x, y) ± g(x, y)
2. (f g)(x, y) = f (x, y)g(x, y)
f (x,y)
f 
3. g (x, y) = g(x,y) , provided that g(x,y) does not assume the value 0.

The formulas for functions of three variables are analogous.

20
3 Applied Math. II

Definition 9 If f is a function of two variables and g is a function of a single


variable, then the composition function gof is defined by

(gof )(x, y) = g(f (x, y))

for all (x,y) in the domain of f .

3.1.2 Graphs
The graph of a function f of two variables is the collection of points (x, y, f (x, y))
for which (x,y) is in the domain of f . Let z = f (x, y), then the graph of f consists
of all points (x,y,z) such that z = f (x, y).

Example 41 Sketch the graph of f (x, y) = x2 + y 2 .

Solution 1 The surface z = x2 + y 2 is a circular parabolic and the traces in the


planes z = k > 0 are circles, while the traces in the planes x = k and y = k are
parabolas.

p
Example 42 Find and describe in graphical terms the domain of g(x, y, z) = 9 − x2 − y 2 − z2 .
p
Solution 2 To define g(x, y, z) = 9 − x2 − y 2 − z2 it should to have 9−x2 −y 2 −z2 ≥
0 or x2 + y 2 + z2 ≤ 9. Therefore the domain of g is a sphere of radius 3 centered at
the origin and its interior.

p
Figure 3.1: f = 9 − x2 − y 2 − z2

By Brhane Aregawi Math Department, MU


21
3 Applied Math. II

3.1.3 Level Curves


In sketching the graph of a function f of two variables, it is often helpful to
determine the intersections of the graph of f with planes of the form z = k. We
call each such intersection the trace of the graph of f in the plane z = k. Thus
the trace of the graph of f in the plane z = k is the collection of points (x, y, k)
such thatf (x, y) = k.

Definition 10 The level curves of a function f of two variablf and it shows the
height value of f at (x, y) from the xy-plane. The level curve f (x, y) = k are just the

Figure 3.2: Level Curves

traces of the graph of f in the horizontal plane z = k projected down to the xy plane
and the trace of f in the plane z = k either lies directly above or directly below the
level curve f (x, y) = k or coincides with it.

Example 43 Sketch the level curves of the function f (x, y) = 6 − 3x − 2y for the
values k = −6, 0, 6, 12.

Solution 3 The level curves are 6 − 3x − 2y = k, this is a family of lines with slope
−3
2 . The four particular level curves with k = −6, 0, 6 and 12 are

Figure 3.3: Level Curves

3x + 2y − 12 = 0
3x + 2y − 6 = 0
3x + 2y = 0 and
3x + 2y + 6 = 0

By Brhane Aregawi Math Department, MU


22
3 Applied Math. II

The level curves are equally spaced parallel lines because the graph of f is a plane.
The contour map of f (x, y) = 6x − 3y − 2y is given below.

Figure 3.4: The level curves of f (x, y) = 6x − 3y − 2y

p
Example 44 Sketch the level curves of the function f (x, y) = 9 − x2 − y 2 for k =
0, 1, 2, 3.
p
Solution 4 The level curves are 9 − x2 − y 2 = k or x2 +y 2 = 9−k 2 . This is a family

of concentric circles with center(0, 0) and radius 9 − k 2 . The casesk = 0, 1, 2, 3.

p
Figure 3.5: The level curves of f (x, y) = 9 − x2 − y 2

Example 45 The level curve(contour curve) the graph of f (x, y) = x2 + y 2 is the set
of circles given by

Figure 3.6: The level curves of f (x, y) = x2 + y 2

By Brhane Aregawi Math Department, MU


23
3 Applied Math. II

3.2 Limit and Continuity


3.2.1 Limit
Definition 11 Let f be defined on the interior of a circle centered at the point (a,b)
except possibly at (a,b) it self. We say that lim f (x, y) = L if for every  > 0,
(x,y)→(a,b)
there is a δ > 0 such that
p
|f (x, y) − L| <  whenever 0 < (x − a)2 + (y − b)2 < δ.

Example 46 Verify that lim x = a and lim y = b.


(x,y)→(a,b) (x,y)→(a,b)

Solution 5 Certainly, both of these limits are intuitively quite clear, we can use the
above definition to verify them, however. Given any number  > 0, we must find
another number δ > 0 so that
p
|x − a| <  whenever 0 < (x − a)2 + (y − b)2 < δ
Notice that
p p
(x − a)2 + (y − b)2 ≥ (x − a)2 = |x − a|
and so, taking δ = , we have that
p p
|x − a| = (x − a)2 ≤ (x − a)2 + (y − b)2 < 
p
whenever 0 < (x − a)2 + (y − b)2 < δ. Lick wise, we can show that lim y = b.
(x,y)→(a,b)

Definition 12 The function f has a limit L at the point (a,b), written

lim f (x, y) = L
(x,y)→(a,b)

if the deference |f (x, y) − L| is as small as we wish whenever the distance from the
point (x,y) to the point (a,b) is sufficiently small, but not zero.
Note that: The limit of a function of two variables (if exists) is unique.

Operation of Limit
If f (x, y) and g(x, y) have limits at (x,y) approaches to (a,b), respectively; that
is,
lim f (x, y) = K and lim g(x, y) = L
(x,y)→(a,b) (x,y)→(a,b)

then
• lim [f (x, y) ± g(x, y)] = K ± L
(x,y)→(a,b)

• lim [f (x, y)ġ(x, y)] = K L̇


(x,y)→(a,b)
" #
f (x, y) K
• lim = , provided L , 0
(x,y)→(a,b) g(x, y) L

By Brhane Aregawi Math Department, MU


24
3 Applied Math. II

• lim cf (x, y) = cK, where c is some constant number.


(x,y)→(a,b)

xsiny+x +2 2
Example 47 Evaluate the limit of the function f (x, y) = √ 2 2 at (x,y) ap-
x +y +1
proaches to (0,0).

Solution 6
xsiny + x2 + 2
lim f (x, y) = lim p
(x,y)→(0,0) (x,y)→(0,0) x2 + y 2 + 1
lim (xsiny + x2 + 2)
(x,y)→(0,0)
= q
lim x2 + y 2 + 1
(x,y)→(0,0)

=2

2x2 y + 3xy
Example 48 Evaluate lim .
(x,y)→(2,1) 5xy 2 + 3y

Solution 7 First, note that this limit of rational function is the quationt of the two
polynomials. Since the limit in the denominator is
lim (5xy 2 + 3y) = 10 + 3 = 13 , 0
(x,y)→(2,1)

we have
lim (2x2 y + 3xy)
2x2 y + 3xy (x,y)→(2,1)
lim =
(x,y)→(2,1) 5xy 2 + 3y lim (5xy 2 + 3y)
(x,y)→(2,1)
14
=
13
Remark 1 1. If f (x, y) approaches L1 as (x,y) approaches (a,b) along a path p1
and f (x, y) approaches L2 , L1 as (x,y) approaches (a,b) along a path p2 , then
lim f (x, y) does not exist. So, the following paths are the simplest paths
(x,y)→(a,b)
to check the existence of limit
(a) Vertical line test (x = a, y → b)
b) Horizontal line test (y = b, x → a)
c) y = g(x), x → a where b = g(a) and
d) x = g(y), y → b where a = g(b)
2. Use polar coordinate to check the existence of limit. Consider (x,y) approaches
to (a,b). We can convert it by polar coordinate in the form x = a + rcosθ and
y = b + rsinθ. So,
lim f (x, y) = lim f (r, θ)
(x,y)→(a,b) r→0

If lim f (r, θ) exist; that is, constant number and unique for any θ, then the
r→0
limit exist. Otherwise, the limit does not exist.

By Brhane Aregawi Math Department, MU


25
3 Applied Math. II

Figure 3.7: f approaches from (x,y) to (a,b) in different paths

y
Example 49 Evaluate lim .
(x,y)→(1,0) x + y − 1

Solution 8 First, we consider the vertical line path along the line x = 1 and com-
pute the limit as y approaches 0. If (x, y) → (1, 0) along the line x = 1, we have
y
lim = lim 1 = 1
(x,y)→(1,0) x + y − 1 y→0

Next consider the horizontal line y = 0 and compute the limit as x approaches 1,
then we have
y
lim = lim 0 = 0
(x,y)→(1,0) x + y − 1 x→1
Since the function is approaching two different values along two different paths to
the point (1,0), the limit does not exist.
xy
Example 50 Show that lim doesn’t exist.
(x,y)→(0,0) x2 + y 2

xy
Solution 9 To show that lim doesn’t exist, it suffice to show that the
(x,y)→(0,0) x2 + y 2
limits doesn’t exist at different direction of approaching the point (0,0). Suppose,

Figure 3.8: f approaches from (0,0) along different paths

we approach (0,0) through the parabola y = x2 . Then,

xy x3
lim = lim  =0
(x,y)→(0,0) x2 + y 2

(x,y)→(0,0) x3 x + 1x

By Brhane Aregawi Math Department, MU


26
3 Applied Math. II

On the other hand, approaching (0,0) through the line y = x. we get,

xy x2 1
lim = lim =
(x,y)→(0,0) x2 + y 2 (x,y)→(0,0) x2 + x2 2

Hence, since
xy xy
lim , lim
(x,x2 )→(0,0) x2 + y 2 (x,x)→(0,0) x2 + y 2
xy
We conclude that lim doesn’t exist.
(x,y)→(0,0) x2 + y 2

xy 2
Example 51 Evaluate lim .
(x,y)→(0,0) x2 + y 4

Solution 10 First we consider the path x = 0 and get

0y 2
lim = lim 0 = 0
(0,y)→(0,0) 02 + y 4 y→0

Next, following the path y = 0, we get

x02
lim = lim 0 = 0
(x,0)→(0,0) x2 + 04 x→0

Since the limits along the first two paths are the same, we try another path. The
next most obvious choice of path through (0,0) is the line y = x. As it turns out, this
limit is
xx2 x
lim = lim =0
(x,x)→(0,0) x2 + x4 x→0 1 + x2

also, so we will need to try yet another path. Path y = kx Finally, notice that along
the path x = y 2 , the terms x2 and y 4 will be equal. We then have

y2y2 y4
lim = lim =1
(y 2 ,y)→(0,0) (y 2 )2 + y 4 y4

Since this limits does not agree with the limits along the earlier paths, the original
limit does not exist.

xy 2
Figure 3.9: f (x, y) = x2 +y 4

Example 52 Determine whether lim (x2 + y 2 ) ln(x2 + y 2 ) exist or not by using


(x,y)→(0,0)
polar coordinate.

By Brhane Aregawi Math Department, MU


27
3 Applied Math. II

Solution 11 Let x = r cos θ and y = r sin θ, then


lim (x2 + y 2 )ln(x2 + y 2 ) =lim r 2 ln(r 2 ), but lim r 2 ln(r 2 ) = 0 · ∞ type
(x,y)→(0,0) r→0 r→0
ln r 2
= lim 1
r→0
r2
2r
r2
= lim −2 , by L’Hopital’s Rule
r→0 3
r
lim −r 2
r→
=0
Therefore, lim (x2 + y 2 )ln(x2 + y 2 ) exist.
(x,y)→(0,0)

Theorem 14 Suppose that |f (x, y) − L| ≤ g(x, y) for all (x,y) in the interior of some
circle centered at (a,b) except possibly at (a,b). If lim g(x, y) = 0, then
(x,y)→(a,b)

lim f (x, y) = L
(x,y)→(a,b)

x2 y
Example 53 Evaluate lim .
(x,y)→(0,0) x2 + y 2

Solution 12 As we did in the earlier example, we start by looking at the limit along
several paths through (0,0). Along the path x = 0, we have
0y
lim = 0.
(0,y)→(0,0) 0 + y 2

Along the path y = 0, we have


x2 0
lim = 0.
(x,0)→(0,0) x2 + 0

Further, along the path y = x, we have


x3 x
lim = 2 2
= lim = 0
(x,x)→(0,0) x +x x→0 2

At this stage, we know that if the limit exists, it must equal 0. We could try other
paths, but our last calculation gives an important clue that the limit does not exist.
In that calculation, after simplifying the expression, there remained an extra power
of x in the numerator forcing the limit to 0. To show that the limit equals L = 0,
consider
x2 y
|f (x, y) − L| = |f (x, y) − 0| = 2
x + y 2
Notice that if there was no y 2 term in the denominator, we could cancel the x2 terms.
Since x2 + y 2 ≥ x2 ,we have for x , 0.

x2 y x2 y
|f (x, y) − L| = |f (x, y) − 0| = 2 ≤ = |y|
x + y 2 x2

x2 y
certainly, lim |y| = 0 and so the above theorem gives as lim = 0.
(x,y)→(0,0) (x,y)→(0,0) x2 + y 2

By Brhane Aregawi Math Department, MU


28
3 Applied Math. II

3.2.2 Continuity
Definition 13 Suppose that f (x, y) is defined in the interior of a circle centered at
the point (a,b). We say that f is continuous at (a,b) if

lim f (x, y) = f (a, b)


(x,y)→(a,b)

If f (x, y) is not continuous at (a,b), then we call (a,b) a discontinuity of f .

If a function f is continuous at each point in its domain, then we will refer to


f as a continuous function. For example, if f (x, y) = x and g(x, y) = y, then f
and g are continuous functions.

Continuity on a Set
Let R be a set in the plane. Then for each point P in the plane, one of the
following conditions hold
1. There is an open disk centered at P and contained in R. In this case P is
an interior point of R.

2. There is an open disk centered at P and containing no points of R. In this


case P is an exterior point of R.
3. Every open disk centered at P contains a point in R and a point outside of
R. In this case P is an boundary point of R. If a set R contains its boundary,
then R is closed.

If (a,b) is a boundary point of the domain D of a function f , we say that f is


continuous at (a,b) if
lim f (x, y) = f (a, b), where (x, y) ∈ D
(x,y)→(a,b)

This notation indicates that the limit is taken only along paths lying completely
inside D.

Example 54 Let R be a rectangular region consisting of all points (x,y) such that
0 ≤ x ≤ 1 and 0 ≤ y ≤ 2. Let
(
4 − x − y, f or (x, y) in R;
f (x, y) =
0, f or (x, y) not in R.

Show that f is continous on R but f is not a continous function.

Solution 13 Since the polynomial 4 − x − y is continous, both the polynomial and


f are continous on R. Next, since |f (x, y)| ≤ 1 for all (x,y) in R and f (x, y) = 0 for
(x,y) not in R, it follows that f has no limit at any boundary point of R and thus is
not continous.

Theorem 15 If f (x, y) and g(x, y) are continuous at (a,b), then f + g, f − g and


f
f g are all continuous at (a,b). Further g , provided that g(a, b) , 0 is continuous at
(a,b).

By Brhane Aregawi Math Department, MU


29
3 Applied Math. II

Theorem 16 All polynomials are continuous everywhere.

x2 +y3
Example 55 The function defined by f (x, y) = x3 y+4 is continuous at the point
(0,0) because
lim f (x, y) = f (0, 0)
(x,y)→(0,0)

Theorem 17 Suppose that f (x, y) is continuous at (a,b) and g(x) is continuous at


the point f (a, b). Then h(x, y) = (gof )(x, y) = g(f (x, y)) is continuous at (a,b).
2y
Example 56 Show that f (x, y) = ex is continuous.

Solution 14 f (x, y) = g(h(x, y)), where g(t) = et and h(x, y) = x2 y. Since g is con-
tinuous for all values of t and h is a polynomial in x and y ( and hence continuous
for all x and y). It follows from the above theorem that f is continuous for all x and
y.

x
Example 57 Find all points where the given function is continuous f (x, y) = x2 −y
.

Solution 15 Since f (x, y) is a quotient of two polynomials and so it is continuous


at any point where we do not divide by 0. Since division by zero occurs only when
y = x2 . We have that f is continuous at all points (x,y) with y , x2 .

Example 58 Find all points where the given function is continuous


 4
 x(x2x+y 2 ) if (x, y) , (0, 0);


g(x, y) = 
 0
 if (x, y) = (0, 0).

Solution 16 Since the function g(x, y) is a quotient of polynomials, except at the


origin. Notice that there is a division by zero whenever x = 0. We must consider the
point (0,0). Separately, however, since the function is not defined by the rational
expression there. We can verify that

lim g(x, y) = 0
(x,y)→(0,0)

Using the following string of inequalities. Notice that for (x, y) , (0, 0).

x4 x4
|g(x, y)| = |g(x, y) − 0| = ≤ = |x|
x(x2 + y 2 ) x(x2 )

and |x| → 0 as (x, y) → (0, 0), we have that

lim g(x, y) − 0 = g(0, 0)


(x,y)→(0,0)

So that g is continuous at (0,0). Putting this all together, we get that g is continuous
at the origin and also at all points (x,y) with x , 0.

By Brhane Aregawi Math Department, MU


30
3 Applied Math. II

Definition 14 Suppose that f (x, y, z) is defined in the interior of a sphere centered


at the point (a,b,c). We say that f is continuous at (a,b,c) if

lim f (x, y, z) = f (a, b, c)


(x,y,z)→(a,b,c)

If f (x, y, z) is not continuous at (a,b,c), then we call (a,b,c) a discontinuity of f .

x2 + y 2 − z2
Example 59 Evaluate lim .
(x,y,z)→(0,0,0) x2 + y 2 + z2

Solution 17 First, we consider the path x = y = 0 (the z-axis). There, we have


02 + 02 − z2
lim = −1 Along the path x = z = 0 (the y-axis), we have
(0,0,z)→(0,0,0) 02 + 02 + z2
02 + y 2 − 02 y2
lim = lim = 1 Since the limits along these two specific
(0,y,0)→(0,0,0) 02 + y 2 + 02 y→0 y 2
paths do not agree, the limits does not exist.

3.3 Partial Derivatives; Higher Order Partial Deriva-


tives
3.3.1 Partial Derivatives
∂f
Definition 15 • The partial derivative of f (x, y) with respect to x written ∂x
,
is defined by
∂f f (x + h, y) − f (x, y)
= lim
∂x h→0 h
for any values of x and y for which the limit exists.
∂f
• The partial derivative of f (x, y) with respect to y written ∂y
, is defined by

∂f f (x, y + h) − f (x, y)
= lim
∂y h→0 h

for any values of x and y for which the limit exists.

Remark 2 • Prime notation; that is, f 0 (x, y) is not applicable in several vari-
ables.
∂f ∂
• The notation of partial derivatives is represented by ∂x
(x, y) = fx (x, y) = ∂x f (x, y).
∂f ∂
Similarly, we have ∂y
(x, y) = fy (x, y) = ∂x f (x, y).

∂f
• To compute the partial derivative of f (x, y) respect to x; that is, ∂x simply
take an ordinary derivative with respect to x, while treating y as a constant.
∂f
Similarly, we can compute ∂y by taking an ordinary derivative with respect
to y, while treating x as a constant.

Example 60 Let f (x, y) = x2 + y 2 . Find the partial derivatives of f at (0,0).

By Brhane Aregawi Math Department, MU


31
3 Applied Math. II

Solution 18 By definition

∂f f (0 + h, 0) − f (0, 0)
= lim = lim h = 0
∂x h→0 h h→0

∂f
Similarly, ∂x
(0, 0) = 0.

Theorem 18 If f is differentiable at (x0 , y0 ), then f is continuous at (x0 , y0 ).


2
Example 61 Evaluate the partial derivatives of f (x, y) = exy cosy.

∂f ∂f
Solution 19 We have to compute ∂x
and ∂y
respectively as
∂f 2
∂x
(x, y) = exy (y 2 cos y)
2
= y 2 (cos y)exy
∂f 2 2
∂y
(x, y) = (−siny)exy + 2xy cos yexy
2
= exy (−siny + 2xy cos y)

∂f ∂f ∂f
Example 62 For f (x, y) = 3x2 +x3 y +4y 2 . Compute ∂x
(x, y), ∂y (x, y), ∂x (1, 0) and
∂f
∂y
(2, −1).

∂f
Solution 20 Compute ∂x
by treating y as a constant. We have

∂f ∂
∂x
(x, y) = ∂x (3x2 + x3 y + 4y 2 )
= 6x + (3x2 )y + 0
= 6x + 3x2 y

Notice here that the partial derivative of 4y 2 with respect to x is 0, since 4y 2 is


treated as if it were a constant when differentiating with respect to x.
∂f
Next we compute ∂y by treating x as a constant. We have

∂f ∂f
∂y
(x, y) = ∂y (3x2 + x3 + 4y 2 )
= 0 + x3 (1) + 8y
= x3 + 8y

Substituting values in for x and y, we get

∂f
fx (1, 0) = (1, 0) = 6 + 0 = 6
∂x
∂f
fy (2, −1) = (2, −1) = 8 − 8 = 0
∂y

By Brhane Aregawi Math Department, MU


32
3 Applied Math. II

3.3.2 Higher Order Derivative


For functions of two variables, there are four different-second order partial
derivatives.Those are
∂ ∂f
 
• ∂x ∂x
= fxx
 
∂ ∂f
• ∂y ∂y
= fyy
 
∂ ∂f ∂2 f
• ∂x ∂y
= ∂x∂y
= fxy , mixed 2nd order partial derivatives.

∂2 f
 ∂f 

• ∂y ∂x
= ∂y∂x
= fyx mixed 2nd order partial derivatives.

Example 63 Find all second-order partial derivative of f (x, y) = x2 y − y 3 + ln x.

Solution 21 We start by computing the first-order partial derivatives


∂f ∂f
∂x
= 2xy + 1x and ∂y
= x2 − 3y 2

we then have
∂2 f
 ∂f 

∂x2
= ∂x  ∂x 
= ∂
∂x
2xy + 1x = 2y − x12
∂2 f
 ∂f 

∂y∂x
= ∂y  ∂x 
= ∂
∂y
2xy + 1x = 2x
 
∂2 f ∂ ∂f
∂x∂y
= ∂x ∂y

= ∂x
(x2 − 3y 2 ) = 2x
and finally  
∂2 f ∂ ∂f
∂y 2
= ∂y ∂y

= ∂y
(x2 − 3y 2 ) = −6y

Theorem 19 If fxy (x, y) and fyx (x, y) are continuous on an open set containing
(a,b), then fxy (a, b) = fyx (a, b).

Example 64 fxy (x, y) and fyx (x, y) of f (x, y) = x2 y − y 3 + ln x are equal with in the
domain of f (x, y). Partial derivatives of order 3 or higher can also be defined in
similar ways as second order. For instance,
∂3 f
 
fxyy = fxy y = ∂y 2 ∂x and fxyy = fyxy = fyyx if these functions are continuous.

Example 65 For f (x, y) = cos(xy) − x3 + x4 . Compute fxyy and fxyyy .

By Brhane Aregawi Math Department, MU


33
3 Applied Math. II

Solution 22 we have

fx (x, y) = ∂x (cos(xy) − x3 + x4 )
= −y sin(xy) − 3x2
Differentiating fx with respect to y gives us
∂ ∂f
fxy (x, y) = ∂y ∂x

= ∂y
(−y sin(xy) − 3x2 )
= − sin(xy) − xy cos(xy)
and  
∂ ∂f


fxyy (x, y) = ∂y ∂y ∂x

= ∂y (− sin(xy) − xy cos(xy))
= −2x cos(xy) + x2 y sin(xy)
finally, we have
  
∂ ∂f

∂ ∂
fxyyy (x, y) = ∂y ∂y ∂y ∂x

= ∂y (−2x cos(xy) + x2 y sin(xy))
= 3x2 sin(xy) + x3 y cos(xy)

3.4 Tangent Line


The vertical plane y = y0 ; that is, {(x, y, z)|y=y0 } intersects the graph of f in a
curve C which is essentially the graph of f (x, y0 ) considered as a function of one
variable x. Then, fx (x0 , y0 ) is the derivative of the function k(x) = f (x, y0 ) and
hence, fx (x0 , y0 ) is the slope of the tangent line to the curve C at (x0 , y0 , f (x0 , y0 )).
Similarly, fy (x0 , y0 ) is the slope of the tangent line to the graph of f (x0 , y) at
(x0 , y0 , f (x0 , y0 )).

Example 66 A point move along the intersection of the graph of z = x2 + 3y 2 and


the plane x = 2. Find the rate at which z changes with y when the point is at (2,1,7).
∂z
Solution 23 First we have to find ∂y

∂z ∂ 2
= (x + 3y 2 ) = 6y.
∂y ∂y
∂z
Now we can evaluate ∂y
at a point (2,1,7)

∂z
(2, 1) = 6(1) = 6
∂y
Therefore, the rate at which z changes with y when the point is at (2,1,7) is 6.

Example 67 Find the slope of the tangent line at (2,1,2) to the curve of intersection
of x2 + y 2 + z2 = 9 and the planes x = 2.

Solution 24 Consider x2 +y 2 +z2 = 9. The intersection of the surface x2 +y 2 +z2 = 9


∂z
and the plane x = 2 is the curve y 2 + z2 = 5. Then, ∂y (2, 1, 2) = −1
2 .

By Brhane Aregawi Math Department, MU


34
3 Applied Math. II

3.5 Directional Derivatives and Gradients


3.5.1 Directional Derivatives
Definition 16 The directional derivative of f (x, y) at the point (a,b) in the direc-
tion of the unit vector u = (u1 , u2 ) is given by

(a + hu1 , b + hu2 ) − f (a, b)


Du f (a, b) = lim
h→0 h
provided the limit exists.

Theorem 20 Suppose that f is differentiable at (a,b) and u = (u1 , u2 ) is any unit


vector. Then, the directional derivative of f (x, y) at the point (a,b) in the direction
of the unit vector u = (u1 , u2 ) can write as

Du f (a, b) = fx (a, b)u1 + fy (a, b)u2 .

2 3
Example
√  68 For f (x, y) = x y − 4y , compute Du f (2, 1) for the direction u =
3 1
2 , 2 and v in the direction from (2,1) to (4,0).

Solution 25 Regardless of the direction, we first need to compute the first partial
derivatives
∂f ∂f
∂x
= 2xy and ∂x
= x2 − 12y 2 , then

fx (2, 1) = 4 and fy (2, 1) = −8

First we have to evaluate


 the directional derivative in the direction of u. So, the unit
 √
3 1
vector is given as 2 ,2 and so, from the above theorem we have

Du f (2, 1) = fx (2, 1)u1 + fy (2, 1)u2 (3.1)


√    √
= 4 23 − 8 12 = 2 3 − 4 (3.2)

In particular, this says that the function is decreasing in this direction. Next, we
have to compute the directional derivative in the direction of v. We must first find
the unit vector u in the indicated direction. Observe that the vector from (2,1)
to (4,0) corresponding
 to
 the position vector (2,-1) and so, the unit vector in that
direction is u = √2 , √
−1
, we then have from the above theorem that
5 5

Du f (2, 1) = fx (2, 1)u1 + fy (2, 1)u2 (3.3)


   
= 4 √2 − 8 √ −1
= √16 (3.4)
5 5 5

Notice that this says that the function is increasing rapidly in this direction.

By Brhane Aregawi Math Department, MU


35
3 Applied Math. II

3.5.2 Gradients
Definition 17 The  gradient of f (x, y) is the vector-valued function is given by
∂f ∂f ∂f ∂f
5f (x, y) = ∂x , ∂y = ∂x i + ∂y j provided both partial derivatives exist. We denote
the gradient of a function f by gradf or 5f ( read as delf ).

Theorem 21 If f is a differentiable function of x and y and u is any unit vector,


then
Du f (x, y) = 5f (x, y) · u.

Example 69 For f (x, y) = x2 + y 2 , find Du f (1, −1) for u in the direction of v =


(−3, 4).
 
∂f ∂f
Solution 26 First, note that 5f (x, y) = ,
∂x ∂y
= (2x, 2y). At the point (1,-1), we
 
4
have 5f (1, −1) = (2, −2). A unit vector in the direction of v is u = −3 5 , 5 . The
directional derivative in this direction is then
−3 4 14
   
Du f (1, −1) = fx (2, 1)u1 + fy (2, 1)u2 = 2 −2 = (3.5)
5 5 5

Definition 18 The directional derivative of f (x, y, z) at the point (a,b,c) in the di-
rection of the unit vector u = (u1 , u2 , u3 ) is given by

f (a + hu1 , b + hu2 , c + hu3 ) − f (a, b, c)


Du f (a, b, c) = lim
h→0 h
provided the limit exists.

Definition 19 The gradient of f (x, y, z) is the vector-valued function


!
∂f ∂f ∂f ∂f ∂f ∂f
5f (x, y, z) = , , = i+ j+ k
∂x ∂y ∂z ∂x ∂y ∂z

provided all partial derivatives exist.

Theorem 22 If f is a differentiable function of x,y and z, respectively and u is any


unit vector, then
Duf (x, y, z) = 5f (x, y, z) · u.
We have

Du f (x, y, z) = 5f (x, y, z) · u (3.6)


= k 5 f (x, y, z)kkuk cos θ (3.7)
= k 5 f (x, y, z)k cos θ (3.8)

Where θ is the angle between the vector 5f (x, y, z) and u.

Theorem 23 Suppose f (x, y) is differentiable at (x0 , y0 ). Then, f (x, y) has direc-


tional derivative at any direction.

Remark 3 Let f be a function of three variables x,y and z. Then,

By Brhane Aregawi Math Department, MU


36
3 Applied Math. II

• 5f (x0 , y0 , z0 ) = fx (x0 , y0 , z0 )i + fy (x0 , y0 , z0 )j + fz (x0 , y0 , z0 )k.


• Du f (x, y, z) = 5f (x, y, z) · u, for any unit vector u = u1 i + u2 j + u3 k.

Theorem 24 Suppose f is differentiable at (x0 , y0 ). Then


1. 5f (x0 , y0 ) = 0 implies Du f (x0 , y0 ) = 0 for any vector u.
2. If 5f (x0 , y0 ) , 0, then the directional derivative of f in the direction of 5f (x0 , y0 )
is the largest and it’s value is k 5 f (x0 , y0 )k.
3. If 5f (x0 , y0 ) , 0, then Du f (x, y), regarded as a function of u, attains its max-
imum value when u points in the same direction as 5f (x0 , y0 ).

Example 70 Find the maximum and minimum rates of change of the function
f (x, y) = x2 + y2 at the point (1,3).

Solution 27 We first compute the gradient 5f (x, y) = (2x, 2y) and evaluate it at
the point (1,3)
5f (1, 3) = (2, 6)
From the above theorem, the maximum rate of change of f at (1,3) is

k 5 f (1, 3)k = k(2, 6)k = 40
5f (1,3) (2,6)
and occur in the direction u = k5f (1,3)k = √ = √1 (2, 6). Similarly, the minimum
40 40
rate of change is √
−k 5 f (1, 3)k = −k(2, 6)k = − 40
5f (1,3) (2,6)
which occurs in the direction u = − k5f (1,3)k = − √ = − √1 (2, 6).
40 40

Theorem 25 Suppose f is differentiable at (x0 , y0 , z0 ). Then


1. 5f (x0 , y0 , z0 ) = 0 implies Du f (x0 , y0 , z0 ) = 0 for any vector u.
2. If 5f (x0 , y0 , z0 ) , 0, then the directional derivative of f in the direction of
5f (x0 , y0 , z0 ) is the largest and it’s value is k 5 f (x0 , y0 , z0 )k.
3. If 5f (x0 , y0 , z0 ) , 0, then Du f (x, y, z), regarded as a function of u, attains its
maximum value when u points in the same direction as 5f (x0 , y0 , z0 ).

Example
  71 2 If 2the temperature at the point (x,y,z) is given by T (x, y, z) = 85 +
z
1 − 100 e−(x +y ) . Find the direction from the point (2,0,99) in which the tempera-
ture increases most rapidly.

Solution 28 We first compute the gradient


z z −(x2 +y 2 ) −1 −(x2 +y 2 )
       
−(x2 +y 2 )
5T (x, y, z) = ∂T , ∂T ∂T
,
∂x ∂y ∂z
= −2x 1 − e , −2y 1 − e , e (3.9)
100 100 100
−1 −4 −1 −4
 
5T (2, 0, 99) = e , 0, e
25 100
To find a unit vector in this direction, you can simplify the algebra by canceling
the common factor of e−4 and multiplying by 100. A unit vector in the direction of
1
(-4,0,-1) and also in the direction of 5T (2, 0, 99), is then sqrt17 (−4, 0, −1).

By Brhane Aregawi Math Department, MU


37
3 Applied Math. II

3.6 Tangent Plane and Tangent Plane Approxima-


tion
3.6.1 Total Differential
For a differentials of two variables z = f (x, y), we define the differentials dx and
dy to be independent variables; that is, they can be given any values. Then the
differential, also called the total differential, is defined by

dz = fx (x, y)dx + fy (x, y)dy (3.10)


∂z ∂z
= ∂x
dx + ∂y dy (3.11)

The sum of the partial differentials with respect to all of the independent vari-
ables is the total differential given by

∂y ∂y
dy = dx1 + · · · + dx
∂x1 ∂xn n

where y = f (x1 , x2 , · · · , xn ).

Example 72 Let z = 4x3 y 2 , find the total differential.

Solution 29 Since f (x, y) = 4x3 y2 , and then fx (x, y) = 12x2 y2 and fy (x, y) = 8x3 y.
Therefore
dz = 12x2 y 2 dx + 8x3 ydy.

3.6.2 Tangent Plane


Definition 20 Let f be a differentiable function at a point (x0 , y0 , z0 ) on a level
surface S of f . If 5f (x0 , y0 , z0 ) , 0, the plane through (x0 , y0 , z0 ) whose normal is
5f (x0 , y0 , z0 ) is the plane tangent to S at (x0 , y0 , z0 ). Any vector that is perpendic-
ular to this tangent plane is said to be normal to the level surface S. This means
5f (x0 , y0 , z0 ) is normal to the level surface S force f (x0 , y0 , z0 ) = c at (x0 , y0 , z0 ).

Example 73 Find a normal vector to the surface xy 3 z2 = 4 at (-1,-1,2).

Solution 30 Let f (x, y, z) = xy 3 z2 −4, so 5f (x, y, z) = (y 3 z2 , 3xy 2 z2 , 2xy 3 z). There-


fore the normal vector to the surface at a point (-1,-1,2) is

5f (−1, −1, 2) = (−4, −12, 4).

Suppose f has continuous partial derivatives. An equation of the tangent plane


to the surface z = f (x, y) at the point (x0 , y0 , z0 ) is

z − z0 = fx (x0 , y0 )(x − x0 ) + fy (x0 , y0 )(y − y0)

Alternatively, let g(x, y, z) = z − f (x, y) and let (x,y,z) be any arbitrary point
on a tangent plane. Since 5g(x0 , y0 , z0 ) is the normal vector to the surface at
(x0 , y0 , z0 ), we have

5g(x0 , y0 , z0 ) · (x − x0 , y − y0 , z − z0 ) = 0

By Brhane Aregawi Math Department, MU


38
3 Applied Math. II

V (fx (x0 , y0 ), −fy (x0 , y0 ), 1) · (x − x0 , y − y0 , z − z0 ) = 0


V −fx (x0 , y0 )(x − x0 ) − fy (x0 , y0 )y − y0 ) + z − z0 = 0
V z − z0 = fx (x0 , y0 )(x − x0 ) + fy (x0 , y0 )(y − y0 )
is the equation of the tangent plane to the surface z = f (x, y) at the point
(x0 , y0 , z0 ).

Definition 21 Suppose f is differentiable at P0 = (x0 , y0 ). Then, the equation of the


tangent plane to the graph of f at (P0 , f (P0 )) is given by z = f (P0 ) + (P − P0 ) · 5f (P0 )
or
f (x0 , y0 ) + fx (x0 , y0 )(x − x0 ) + fy (x0 , y0 )(y − y0 ) − z = 0

Theorem 26 The equation of the tangent plane to the surface f (x, y, z) = c at the
point (x0 , y0 , z0 ) is given by

∂f (x, y, z) ∂f (x, y, z) ∂f (x, y, z)


(x − x0 ) + (y − y0 ) + (z − z0 ) = 0.
∂x ∂y ∂z

Example 74 Find the tangent plane to the elliptic paraboloid z = 3x2 + 2y 2 at the
point (1,1,5).

Solution 31 Let f (x, y) = 3x2 + 2y 2 , then

fx (x, y) = 6x and fy (x, y) = 4y, then


fx (1, 1) = 6 and fy (1, 1) = 4
Therefore, the equation of the tangent plane is

z − 5 = 6(x − 1) + 4(y − 1) V z = 6x + 4y − 5.

Example 75 Find the equation of the tangent plane to a surface x2 + 2y 2 + 3z2 at


(4,-1,1).

Solution 32 First we have to find the normal vector to the plane as 5f (x, y, z) =
(2x, 4y, 6z) and the normal vector at (4,-1,1) will be 5f (4, −1, 1) = (8, −4, 6). Let
(x,y,z) be any arbitrary point on a tangent plane. So

5f (4, −1, 1) · (x − 4, y + 1, z − 1) = 0
(8, −4, 6) · (x − 4, y + 1, z − 1) = 0
8(x − 4) − 4(y + 1) + 6(z − 1) = 0
8x − 4y + 6z = 42

Therefore, 8x − 4y + 6z = 42 is the tangent plane to the surface x2 + 2y 2 + 3z2 at


(4,-1,1).

By Brhane Aregawi Math Department, MU


39
3 Applied Math. II

3.6.3 Tangent Plane Approximation


Definition 22 If z = f (x, y), f is differentiable at the point (x0 , y0 ) if 5z can be
expressed in the form
5z = fx (x0 , y0 ) 4 x + fy (x0 , y0 ) 4 y + 1 4 x + 2 4 y
where 1 → 0 and 2 → 0 as (4x, 4y) → (0, 0).
Replace 4f by 4z in the tangent plane equation and then we can rewrite this as
f (x0 + 4x, y0 + 4y) = f (x0 , y0 ) + fx (x0 , y0 ) 4 x + fy (x0 , y0 ) 4 y + 1 4 x + 2 4 y
Thus, if 4x ≈ 0 and 4y ≈ 0, the terms involving 1 and 2 will be small, and we can
approximate f (x0 + 4x, y0 + 4y) as
f (x0 + 4x, y0 + 4y) ≈ f (x0 , y0 ) + fx (x0 , y0 ) 4 x + fy (x0 , y0 ) 4 y
This formula, which is the two-variables, is called the local tangent planer approxi-
mation of f at (x0 , y0 ). When convenient, we can replace 4x and 4y in this equation
by 4x = x − x0 and 4y = y − y0 and then rewrite the equation as
f (x, y) ≈ f (x0 , y0 ) + fx (x0 , y0 )(x − x0 ) + fy (x0 , y0 )(y − y0 )
It’s sometimes hard to use the above definition directly to check the differentiability
of a function, but the next theorem provides a convenient sufficient condition for
differentiability.

Theorem 27 If the partial derivatives fx and fy exist near (x0 , y0 ) and are contin-
uous at (x0 , y0 ),then f is differentiable at (x0 , y0 ).

Example 76 Show that f (x, y) = xexy is differentiable at (1,0) and find its lin-
earization there. Then use it to approximate f (1, 1, −0.1).

Solution 33 The partial derivatives are


fx (x, y) = exy + xyexy fy (x, y) = x2 exy V fx (1, 0) = 1 f y(1, 0) = 1
Both fx and fy are continuous functions, so f is differentiable by the above theorem.
The linearization is
L(x, y) = f (x, y) + fx (1, 0)(x − 1) + fy (1, 0)(y − 0)
= 1 + 1(x − 1) + 1(y)
= x+y
The corresponding linear approximation is
xexy ≈ x + y
So, f (1.1, −0.1) ≈ 1.1 − 0.1 = 1. Compare this with the actual value of
f (1.1, −0.1) ≈ 1.1e−0.11 ≈ 0.98542

Example 77 An equation of the tangent plane to the graph of the function f (x, y) =
3x2 + 2y 2 at the point (1,1,3) is z = 6x + 4y − 5. Therefore,the linear function of two
variables L(x, y) = 6x + 4y − 5 is a good approximation to f (x, y) when (x,y) is near
(1,1).The function L is called the linearizion of f at (1,1) and the approximation
f (x, y) ≈ 6x + 4y − 5
is called a tangent plane approximation of f at (1,1).

By Brhane Aregawi Math Department, MU


40
3 Applied Math. II

3.7 Chain Rule and Implicitly Differentiation


3.7.1 Chain Rule
If z = f (x(t), y(t)), where x(t) and y(t) are differentiable and f (x, y) is a differen-
tiable function of x and y, then
dz d
dt = dt f (x(t), y(t))
∂f ∂f dy
= ∂x
(x(t), y(t)) dx
dt + ∂y
(x(t), y(t)) dt

For a function z = f (x(t), y(t))


• t is independent variables
• x and y are intermediate variables
• z is dependent variable

Example 78 Consider f (x, y) = x2 cos y, where x(t) = t 2 + 2t, y(t) = sin t. Then
df
evaluate dt .
df ∂f ∂f dy
dt = ∂x · dx
dt + ∂y · dt
= (2x cos y)(2t + 2) − (x2 sin y)(cos t)
= 2(t 2 + 2t) cos(sin t)(2t + 2) − (t 2 + 2t)2 sin(sin t) cos t

If z = f (x, y), where f is a differentiable function of x and y and where both x =


x(s, t) and y = y(s, t) have first order partial derivatives. Then we have the chain
rules
dz ∂f dx ∂f dy dz ∂f dx ∂f dy
ds = ∂x ds
+ ∂y ds
and dt = ∂x dt
+ ∂y dt

Example 79 For f (x, y) = x2 ey , x(t) = t 2 − 1 and y(t) = sin t, find the derivative of
g(t) = f (x(t), y(t)).

∂f ∂f dx
Solution 34 We first compute the derivatives ∂x
= 2xey , ∂x
= x2 ey , dt = 2t and
dy
dt = cos t. The chain rule then gives us
∂f ∂f dy
g 0 (t) = ∂x dx
dt + ∂x dt
= 2xey (2t) + x2 ey cos t
= 2(t 2 − 1)esin t (2t) + (t 2 − 1)2esin t cos t

Example 80 Suppose that f (x, y) = exy , x(u, v) = 3u sin v and y(u, v) = 4v2u. For
∂g ∂g
g(u, v) = f (x(u, v), y(u, v)), find the partial derivatives ∂u and ∂u .

∂f ∂f ∂x
Solution 35 We first compute the derivatives ∂x
= yexy , ∂y
= xexy , ∂u
= 3 sin v
∂y
and ∂u
= 4v 2 . The chain rule then gives us
∂g ∂f
∂x ∂f ∂y
∂u
= ∂x ∂u + ∂y ∂u
= ye (3 sin v) + xexy (4v 2 )
xy
2 2 2 2
= (4v 2 u)e12u v sin v (3 sin v) + (3u sin v)e12u v sin v (4v 2 )

By Brhane Aregawi Math Department, MU


41
3 Applied Math. II

∂x
For the partial derivative of g with respect to v, we compute ∂v
= 3u cos v and
∂y
∂v
= 8uv, Here the chain rule gives us

∂g ∂f ∂x ∂f ∂y
∂v
= ∂x ∂v + ∂y ∂v
= ye (3u cos v) + xexy (8uv)
xy
2 2 2 2
= (4v 2 u)e12u v sin v (3u cos v) + (3u sin v)e12u v sin v (8uv)

3.7.2 Implicitly Differentiation


Theorem 28 If the evaluation F(x, y, z) = 0 defines z implicitly as a differentiable
∂z
function of x and y; that is, z = f (x, y) and if Fz , 0, then partialx = − FFx and
z
∂z F
partialy = − Fy .
z

∂z ∂z 2 3
Example 81 Find partialx and partialy , given that F(x, y, z) = xy +z +sin(xyz) = 0.

Solution 36 First, note that using the usual chain rule, we have

Fx = y 2 + yz cos(xyz)
Fy = 2xy + xz cos(xyz) and
Fz = 3z2 + xy cos(xyz)
We now have
∂z Fx y 2 +yz cos(xyz)
partialx = − Fz = − 3z2 +xy cos(xyz) and
∂z Fy 2xy+xz cos(xyz)
partialy = − Fz = − 3z2 +xy cos(xyz)

Theorem 29 (Implicit Function Theorem) If Fx , Fy and Fz are continuous in-


side a sphere containing the point (a,b,c), where F(a, b, c) = 0 and Fz (a, b, c) , 0,
then the equation F(x, y, z) = 0 implicitly defines z as a function of x and y near by
the point (a,b,c).

3.8 Relative Extrema of Function of Two Variables


Definition 23 Let f (x, y) be a function of two variables. Then,
• f (x, y) is said to have relative maximum value at (x0 , y0 ) if and only if there
exists a disk centered at (x0 , y0 ) such that f (x, y) ≤ f (x0 , y0 ) for all (x,y) in the
interior of the disk.
• f (x, y) is said to have relative minimum value at (x0 , y0 ) if and only if there
exists a disk centered at (x0 , y0 ) such that f (x, y) ≥ f (x0 , y0 ) for all (x,y) in the
interior of the disk.

• In either case f (x0 , y0 ) is called a local extreme of f (x, y).

Definition 24 Suppose f is a function of two variables. Let (x0 , y0 ) be in the do-


main of f . Then, (x0 , y0 ) is called the critical point of f if and only if fx (x0 , y0 ) =
fy (x0 , y0 ) = 0 or either fx (x0 , y0 ) or fy (x0 , y0 ) doesn’t exist.

By Brhane Aregawi Math Department, MU


42
3 Applied Math. II

Figure 3.10: Local and absolute extremum values

Figure 3.11: f (x, y) = y 2 − x2

Local extrema can occur at critical points, every critical points need not corre-
spond to a local extreme. For this reason, we refer to critical points as candi-
dates for local extrema.

Theorem 30 If f (x, y) has a local extreme at (x0 , y0 ), then (x0 , y0 ) must be a critical
point of f .

Definition 25 A critical point of a function f that is neither a point of relative


maximumn or a point of minimum is called a saddle point of f .

Example 82 If f has relative extreme at (x0 , y0 ) and fx (x0 , y0 ) and fy (x0 , y0 ) exist,
then fx (x0 , y0 ) = fy (x0 , y0 ) = 0.

Example 83 Consider f (x, y) = y 2 − x2 . Then, we see that fx (x, y) = fy (x, y) = 0


implies that x = 0 and y = 0. For any circular disk centered at (0,0) in the xy-plane,
we can easily see that f is positive on the y-axis and f is negative on the x-axis.
This shows that the point (0,0) is neither a point of relative maximum nor a point
of relative minimum of f .

Theorem 31 (Second Derivatives Test) Let f be a function of two variables with


continuous second-order partial derivatives in some disk centered at the critical
point (x0 , y0 ). Let
D(x0 , y0 ) = fxx (x0 , y0 ) · fyy (x0 , y0 ) − (fxy (x0 , y0 ))2

By Brhane Aregawi Math Department, MU


43
3 Applied Math. II

1. If D > 0 and f xx(x0 , y0 ) > 0, then f has relative minimum at (x0 , y0 ).


2. If D > 0 and fxx (x0 , y0 ) < 0, then f has relative maximum at (x0 , y0 ).

3. If D < 0, then f has saddle point at (x0 , y0 ).


4. If D = 0, no conclusion can be drawn.

Example 84 Consider f (x, y) = x2 + xy − 2y − 2x + 1. Find all the critical points


and determine the relative maximum and relative minimum values of f .

Solution 37 Let (x,y) be a relative maximum or relative minimum point of f .


Then, we get fx (x, y) = 0 and fy (x, y) = 0; that is, we get the system of equations
with solution (2,-2). Thus, (2,-2) is the only critical point of f .
Now, it is easy to see that D < 0. Hence, by the second partial test, we conclude that
(2,-2)is a saddle point of f .

Example 85 Locate all the critical points and find all the points of relative maxi-
mum and relative minimum of f (x, y) = 2x2 − y 3 − 2xy.

Solution 38 We first compute the first partial derivatives

fx = 4x − 2y and fy = −3y 2 − 2x
Since both partial derivatives are defined for all (x,y), the critical points are solutions
of the two equations.
fx = 4x − 2y = 0 and fy = −3y 2 − 2x = 0

Solving the first equation for y, we get y = 2x. Substituting this in to the second
equation, we have
0 = −3(4x2 ) − 2x
= −12x2 − 2x
= −2x(6x + 1)
So that x = 0 or x = −1
6 .
The corresponding y-values are y = 0 and y = −1 3 . The only two critical points are
 
−1 −1
then (0,0) and 6 , 3 . To classify these points, we first compute the second partial
derivatives
fxx = 4, fyy = −6y and fxy = −2
then test the discriminant. We have

 0) = (4)(0) − (−2)2 = −4
D(0,
D −1 −1
6 , 3 = (4)(2) − (−2)2 = 4

From the above theorem. We conclude that there is a saddle


 point of f at (0,0),since

D(0, 0) < 0. Further, there is a local minimum at −1
6 , −1
3 . Since D −16 , −1
3 > 0 and
 
−1 −1
fxx 6 , 3 > 0.

By Brhane Aregawi Math Department, MU


44
3 Applied Math. II

3.9 Largest and Smallest Values of a Function on a


Given Set
Definition 26 Let f (x, y) be a function of two variables. Then
• f (x, y) is said to have absolute maximum at (x0 , y0 ) if and only if f (x, y) ≤
f (x0 , y0 ) for all (x,y) in the domain of f .
• f (x, y) is said to have absolute minimum at (x0 , y0 ) if and only if f (x, y) ≥
f (x0 , y0 ) for all (x,y) in the domain of f .

Theorem 32 If f (x, y) is continuous in a closed and bounded subset S of the plane,


then f has both absolute maximum(minimum)value in S. Further, the absolute ex-
trema must occur at either at a critical point in R or on the boundary of R.

Remark 4 The following steps helps us to find absolute extreme values of f in a


bounded region S.
• Step 1: Find critical points that lie in the interior of S.
• Step 2: Obtain the maximum(minimum) value of f on the boundary of S.
• Step 3: Compare the values of f obtained in step 1 and step 2.

Example 86 Let f (x, y) = xy−x2 and let R be the square region in the first quadrant
having vertex (0,0), (1,0), (1,1) and (0,1). Find the extreme values of f on R.

Solution 39 By the Maximum-minimum Theorem f has extreme values on R.


Since
fx (x, y) = y − 2x and fy (x, y) = x
it follows that fx (x, y) = 0 if y = 2x and fy (x, y) = 0 if x = 0. Thus the only critical
point of f is (0,0), which happens to be a boundary point of R. Therefore the extreme
values of f on R must occur on the boundary of R, which is composed of the four
line segments. On the first line segment, x = 0 and 0 ≤ y ≤ 1, and since f (0, y) = 0,
the maximum and minimum values of f are both 0. On the second line segment,
y = 0 and 0 ≤ x ≤ 1, and since
f (x, 0) = −x2
the maximum value of f on this line is 0 and the minimum value of f is -1. On the
third line segment, x = 1 and 0 ≤ x ≤ 1, and since
f (1, y) = y − 1
the maximum value of f on this line segment is 0 and the minimum value is -1.
Finally, on the fourth line segment, y = 1 and 0 ≤ x ≤ 1, and since
f (x, 1) = x − x2
the maximum value of f is 14 and the minimum value is 0. By comparing the ex-
treme values, we conclude that the maximum value of f on R is 14 and the minimum
value is -1.

Example 87 Consider the function f (x, y) = x2 + y 2 . It can be easily shown that f


has an absolute minimum at the origin.(see the graph below).

By Brhane Aregawi Math Department, MU


45
3 Applied Math. II

Figure 3.12: f (x, y) = x2 + y 2

3.10 Extreme Value under Constraint Condition


3.10.1 Method of Lagrange Multipliers for One Constraint
Consider the problem
Maximum(minimum) of f (x, y, z)
subject to g(x, y, z) = 0

(assuming that these extreme values exist and 5g , 0 on the surface g(x, y, z) =
0)
• Find all values of x,y,z and λ such that 5f (x, y, z) = λ 5 g(x, y, z) and
g(x, y, z) = 0.

• Evaluate f at all points (x,y,z) that result from (a).


The largest of these values is the maximum value of f ; the smallest is the
minimum value of f .

Example 88 Find the maximum and minimum of the function f (x, y) = x2 + y 2 on


the ellipse x2 + 2y 2 = 1.

Solution 40 Let g(x, y) = x2 + 2y 2 − 1 and suppose (x,y) is a point of extreme value


of f on the ellipse. Then, there exists a real number λ such that

5f (x, y) = λ 5 g(x, y)

This is equivalent to solving

2x = λ(2x) 2y = λ(4y)

for x,y and λ. Since (x,y) is on the ellipse, both x and y can not be zero at the same
time. Hence, we get either x = 0, λ = 21 or y = 0, λ = 1. Since we seek points on the
ellipse, x = 0 implies y = ± √1 and y = 0 implies that x = ±1. Moreover,we see that
  2
1 1
f 0, ± √ = 2 , f (±1, 0) = 1. We conclude that if f has maximum, then it occurs at
2  
(±1, 0) and if it has minimum, then it occurs at 0, ± √1 .
2

By Brhane Aregawi Math Department, MU


46
3 Applied Math. II

Example 89 Find the point on the line y = 3 − 2x that is closest to the origin.

Solution 41 the distance from the origin to the point (x,y) on the line is expressed
as q
h(x, y) = x2 + y 2

but before we continue, we must point out that the distance is minimized at exactly
the same point at which the square of the distance is minimized. Notice that if we
instead minimize the square of the distance, given by

f (x, y) = x2 + y 2

for f (x, y) = x2 +y 2 , we have 5f (x, y) = (2x, 2y) and for g(x, y) = 2x +y −3, we have
5g(x, y) = (2, 1). The vector equation 5(x, y) = λ 5 g(x, y) becomes

(2x, 2y) = λ(2, 1)

from which it follows that


2x = 2λ and 2y = λ

Solving the second equation for λ, we have λ = 2y. The first equation then gives us
x = λ = 2y. Substituting x = 2y in to the constraint equation y = 3 − 2x, we have
y = 3 − 2(2y) or 5y = 3
 
The solution is y = 35 , giving us x = 2y = 65 . The closest point is then 6 3
5, 5 .

Example 90 A rocket is launched with a constant thrust corresponding to an ac-


ft
celeration of u s2 . Ignoring air resistance, the rocket’s height after t second is given
by f (t, u) = 21 (u − 32)t 2 f eet. Fuel usage for t seconds is proportional to u 2 t, so
the limited fuel capacity of the rocket can be expressed by an equation of the form
u 2 t = 10, 000. Find the value of u that maximized the height that the rocket reaches.

Solution 42 Now we have to find solutions of 5f (t, u) = λ 5 g(t, u) where g(t, u) =


u 2 t − 10, 000 = 0 is the constraint equation. We have

5f (t, u) = λ 5 g(t, u) and 5g(t, u) = (u 2 , 2ut)


Now we must have

1
((u − 32)t, t 2 ) = λ(u 2 , 2ut)
2
For some constant λ. It follows that
(u − 32)t = λu 2 and 21 t 2 = λ2ut
solving both equations for λ, we have

(u − 32)t 1 2 2
λ= = t u
u2 2

By Brhane Aregawi Math Department, MU


47
3 Applied Math. II

This gives us
2u(u − 32)t 2 = t 2 u 2
Solution include t = 0 and u = 0 on physical ground, we can argue that these solu-
tions represent minimum heights, since zero thrust or zero time would correspond to
zero height. Canceling the factors t2 and u, we have 4(u − 32) = u. The solution to
10,000 10,000
this is u = 128
3 , with this value of u, the engines can burn t = 2u 2 = 128 2 ≈ 5.5s
  ( 3 )
1 128 2
with the rocket reaches height z = 2 3 − 32 (5.5s) ≈ 161f eet.

Example 91 Suppose that the temperature of a metal plate is given by T (x, y) =


x2 + 2x + y 2 , for points (x,y) on the elliptical plate defined by x2 + 4y 2 ≤ 24. Find
the maximum and minimum temperature on the plate.

Solution 43 The plate corresponds to the region R on side x2 + 4y 2 ≤ 24. We first


look for critical points of T(x,y) in side the region R, we have 5T (x, y) = (2x+2, 2y) =
(0, 0) if x = −1 and y = 0. At this point, T (−1, 0) = −1. We next look for the extrema
of T (x, y) on the ellipse x2 + 4y 2 = 24. We first rewrite the constraints equation as
g(x, y) = x2 + 4y 2 − 24 = 0. Now any extrema will satisfy the Lagrange multiplier
equation
5T (x, y) = λ 5 g(x, y) or
(2x, 2y) = λ(2x, 8y) = (2λx, 8λy)
This occurs when 2x + 2 = 2λx and 2y = 8λy
Notice that the second equation hold when y = 0 or λ = 14 . If y = 0, the constraint

x2 + 4y 2 = 24 gives x = ± 24. If λ = 14 ,the first equation becomes 2x + 2 = 21 x so

that x = − 34 . The constraint x2 + 4y 2 = 24 now gives y = ± 350 . We now need only
compare the function values at all these points (the one interior critical point and
the candidates for boundary extrema) as follows

T√(−1, 0) = −1 √
T (√24, 0) = 24 + 2√24 ≈ 33.8
T (− 24, 0) = 24 − 2 24 ≈ 14.2

T − 43 , 350 = 14
3 ≈ 4.7 and
 √ 
T − 43 , −50
3 = 14
3 ≈ 4.7

From this list, it is easy to identify


√ the minimum
√ value of -1 at the point (-1,0) and
the maximum value of 24 + 2 24 at the point ( 24, 0).

Example 92 For a business that produces three products, suppose that when pro-
ducing x,y and z thousand units of the products, the profit of the company (in
thousand of dollars) can be modeled by P (x, y, z) = 4x + 8y + 6z. Manufacturing
constraints force x2 + 4y 2 + 2z2 ≤ 800. Find the maximum profit for the company.

Solution 44 We start with 5P (x, y, z) = (4, 8, 6) and note that there are no critical
points. This says that the extrema must lie on the boundary of the constraint region.
That is, they must satisfy the constraint equation

g(x, , y, z) = x2 + 4y 2 + 2z2 − 800 = 0

By Brhane Aregawi Math Department, MU


48
3 Applied Math. II

Then the Lagrange multiplier equation is

5g(x, y, z) = λ 5 g(x, y, z) or
(4, 8, 6) = λ(2x, 8y, 4z)
= (2λx, 8λy, 4λz)

This occurs when 4 = 2λx, 8 = 8λy and 6 = 4λz From the first equation, we get
2
x = lambda . The second equation gives us y = λ1 and finally the third equation gives
3
us z = 2λ . From the constraint equation x2 + 4y 2 + 2z2 = 800. We know have
 2  2  2
2 1 3 5
800 = +4 +2 =
λ λ 2λ 2λ2
25
so that λ2 = 1600 and then λ = 18 .
The values of x,y and z are positive since they represent a number of quantity, so
since x > 0, we must have λ = 2x > 0.
The only candidate for an extrema is then
2 1 3
x= λ = 16, y = λ = 8 and z = 2λ = 12

Where the corresponding profit is P (16, 8, 12) = 4(14) + 8(8) + 6(12) = 200. Observe
that this is the maximum profit, while λ = − 18 gives the minimum value of the profit
function.
Consider the problem Maximum(minimum) of f (x, y, z) subject to g(x, y, z) ≤ 0
we first find the critical points of f (x, y, z) that satisfy the constraint, then find
the extrema of the function on the boundary g(x, y, z) = 0 (the constraint curve
or set ) and finally, compare the function values.

3.10.2 Method of Lagrange Multipliers for Two Constraints


Consider the problem

Maximum(minimum) of f (x, y, z)
subject to

g(x, y, z) = 0
h(x, y, z) = 0
(assuming that these extreme values exist,5g , a and 5h , 0 on the surface
g(x, y, z) = 0 and h(x, y, z) = 0, respectively and 5g is not parallel with 5h)
• Step 1: Find all values of x,y,z,λ and µ such that

5f (x, y, z) = λ 5 g(x, y, z) + µ 5 h(x, y, z)


g(x, y, z) = 0 and h(x, y, z) = 0

Or
fx (x, y, z) = λgx (x, y, z) + µhx (x, y, z)
fy (x, y, z) = λgy (x, y, z) + λhy (x, y, z)f z(x, y, z) = λgz (x, y, z) + µhz (x, y, z)
g(x, y, z) = 0 and h(x, y, z) = 0

By Brhane Aregawi Math Department, MU


49
3 Applied Math. II

• Step 2: Evaluate f at all points (x,y,z) that result from Step 1. The largest
of these values is the maximum value of f ; the smallest is the minimum
value of f .

Example 93 The plane x + y + z = 12 intersects the paraboloid z = x2 + y 2 in an


ellipse. Find point on the ellipse that is closest to the origin.

Solution 45 We know that the minimizing the distance to the origin is equivalent
to minimizing
f (x, y, z) = x2 + y 2 + z2
We have constraints

g(x, y, z) = x + y + z − 12 = 0 and
h(x, y, z) = x2 + y 2 − z = 0
So,f (x, y, z) = λ 5 g(x, y, z) + µ 5 h(x, y, z) or
(2x, 2y, 2z) = λ(1, 1, 1) + µ(2x, 2y, −1)
Together with the constraint equations, we now have the system of equations.

2x = λ + 2µx
2y = λ + 2µy
2z = λ−µ
x + y + z − 12 = 0 and
x2 + y 2 − z =0

From the above equations (2,2,8) and (-3,-3,18) are the only candidates for extrema.
Finally, since f (2, 2, 8) = 72 and f (−3, −3, 18) = 342. Therefore, the closest point
on the intersection of the two surfaces to the origin is (2,2,8).
By the same reasoning, observe that the furthest point on the intersection of the two
surfaces from the origin is (-3,-3,18). Consider the problem

Maximum(minimum) of f (x, y, z)
subject to

g(x, y, z) ≤ 0
h(x, y, z) ≤ 0
we first find the critical points of f(x,y,z) that satisfy the constraint, then find the ex-
trema of the function on the boundary g(x, y, z) = 0 and h(x, y, z) = 0 (the constraint
curve or set) and finally, compare the function values.

Exercise 5 1. Let f (x, y) = x2 e3xy .


(a) Evaluate f (1, 1)
(b) Find the domain of f
(c) Find the range of f

Ans. 4, R2 , [0, ∞)

z−x2 −y 2
2. Let f (x, y, z) = e .

By Brhane Aregawi Math Department, MU


50
3 Applied Math. II

(a) Evaluate f (2, −1, 6)


(b) Find the domain of f
(c) Find the range of f

Ans. e, (x, y, z)|z ≥ x2 + y 2 , [1, ∞)

y4
3. Find the limit lim x, y) → (0, 0) if it exist.
( x4 + 3y 4

Ans. Does not exist


xy
4. Find the limit lim x, y) → (0, 0) p if it exist.
( x + y2
2

Ans. 0

xy + yz2 + xz2
5. Find the limit lim x, y, z) → (0, 0, 0) if it exist.
( x2 + y 2 + z2

Ans. Does not exist

6. Find√h(x, y) = g(f (x, y)) and the set on which h is continuous, where g(t) =
t 2 + t, f (x, y) = 2x + 3y − 6. p
Ans. h(x, y) = (2x + 3y − 6)2 + 2x + 3y − 6, (x, y)|2x + 3y ≥ 6
7. Determine the set of points at which the function is continuous, where

x2 y 3
2x2 +y 2
if (x, y) , (0, 0)
f (x, y) =
1 if (x, y) = (0, 0)

Ans. (x, y)|(x, y) , (0, 0)

x3 + y 3
8. Use polar coordinate to find the limit lim x, y) → (0, 0) .
( x2 + y 2

Ans. 0
x−y
9. Find the first partial derivatives of the function f (x, y) = x+y .

2y −2x
Ans. fx = x+y , fy = x+y

10. Find the first partial derivatives of the function f (x, y, z) = ln(x + 2y + 3z).

1 2 3
Ans. fx = x+2y+3z , fy = x+2y+3z , fz = x+2y+3z

∂3 f ∂3 f x
11. Find ∂z∂y∂x
and ∂x2 ∂y
for a function f (x, y, z) = y+2z .

12. Find an equation of the tangent plane to the surface z = y cos(x −y) at (2,2,2).

By Brhane Aregawi Math Department, MU


51
3 Applied Math. II

Ans. z = y

13. Find the linearization of the function f (x, y) = x y at (1,4).

Ans. L(x, y) = 2x − 14 y − 1
y
dw
14. Use chain rule to find dt for the functions w = xe z , x = t 2 , y = 1−t, z = 1+2t.
y 2xy
dw
Ans. dt = e z (2t − 2x − z2
)

dz
15. Use chain rule to find dt for the functions z = er cos θ, r = st, θ = s2 + t 2 .

dz s
Ans. dt = er (t cos θ − √ sin θ)
s2 +t 2

16. Find the directional derivative of the function f (x, y) = 1 + 2x x at the point
(3,4) in the direction of v = (3, 4).

23
Ans. 10

17. Find the directional derivative of the function f (x, y) = xy at the point (2,8)
in the direction of v = (5, 4).

2
Ans. 5

18. Find the maximum rate of change of f (x, y) = sin(xy) at the point (1,0) and
the direction in which it occurs.

Ans. 1, (0,1)

19. Find the local maximum and minimum values and saddle point(s) of the func-
tion f (x, y) = x3 − 12xy + 8y 3 .

Ans. Minimum f (2, 1) = −8, saddle point at (0,0)

Maximum(Minimum) of x2 + y 2 + z2
20. Use Lagrange multipliers to find
subject to

x4 + y 4 + z4 − 1 = 0

Ans. Maximum 3, Minimum 1

By Brhane Aregawi Math Department, MU


52
Chapter 4

Multiple Integral

4.1 Double Integrals and Their Evaluation by Iter-


ated Integrals
4.1.1 Definition of Double Integral
Multiple integral is a generalization of the definite integral to functions of
more than one variable, for example, f (x, y) or f (x, y, z). Integrals of a function
of two variables over a region in R2 are called double integrals, and integrals
of a function of three variables over a region of R3 are called triple integrals.

Definition 27 The double integral of f over the rectangular region R is


" m X
X n

f (x, y)dA = lim f (xij , yij∗ )δA
m,n→∞
i=1 j=1

if the limit exist. where the limit is taken so that the number n of subdivisions
increases without limit and such that the largest linear dimension of each approaches
zero.

Example 94 If R = {(x, y) | −1 ≤ x ≤ 1, −2 ≤ y ≤ 2}, evaluate the integral


" √
1 − x2 dA
R

Solution: It would p be √ very difficult to evaluate this integral directly from the
definition but, because
√ 1 − x2 ≥ 0, we can compute the integral by interpreting it
as a volume. If z = 1 − x2 , then x2 + z2 = 1 and z ≥ 0, so the given double integral
represents the volume of the solid S that lies below the circular cylinder x2 + z2 = 1
and above the rectangle R. The volume of S is the area of a semicircle with radius 1
times the length of the cylinder. Thus
" √
1
1 − x2 dA = π(1)2 (4) = 2π
R 2

53
4 Applied Math. II


Figure 4.1: f (x, y) = 1 − x2

4.1.2 Iterated Integral


Theorem 33 (Double integrals as iterated single integrals) Suppose that f (x, y)
is continuous on the rectangle R = [a, b]x[c, d]. Then
" Z b (Z d )
f (x, y)dA = f (x, y)dy dx
a c
Z d (Z b )
= f (x, y)dx dy
c a

Figure 4.2: R = {(x, y) | a ≤ x ≤ b, c ≤ y ≤}

Theorem 34 (Febini’s Theorem) If f is continuous on the rectangle R = {(x, y) |


a ≤ b and c ≤ d, then
" Z bZ d
f (x, y)dA = f (x, y)dydx
a c
Z dZ b
= f (x, y)dxdy
c a

More generally, this is true if we assume that f is bounded on R, f is discontinuous


only on a finite number of smooth curves and iterated integrals exist.
R3R2 R2R3
Example 95 Evaluate 0 1
x2 ydydx and 1 0
x2 ydxdy.

By Brhane Aregawi Math Department, MU


54
4 Applied Math. II

R3R2
Solution 46 For 0 1
x2 ydydx
Z 3Z 2 Z 3 !Z 2
x2 ydydx = x2 ydy dx
0 1 Z03 21 2 !
x y 2
= dx
2 1
Z03
3 2
= x dx
0 2
1 33
2x 0
= 27
2
R2R3
For 1 0
x2 ydxdy
Z 2Z 3 Z 2 Z 3 !
2 2
x ydxdy = x ydx dy
1 0 Z12 0
x3 y
!
3
= 0 dy
3
Z12
= 9ydy
1
9 22
= 2y 1
27
= 2
Z 3Z 2 Z 2Z 3
Here, x2 ydydx = x2 ydxdy.
0 1 1 0
!
Example 96 If R = [0, 2]x[1, 4], evaluate R
(6x2 + 4xy 3 )dA.

Solution 47 We have
! Z 4Z 2

R
(6x2 + 4xy 3 )dA = (6x2 + 4xy 3 )dxdy
Z14 Z0 2 !
2 3
= (6x + 4xy )dx dy
R 41 3 0 2
= ( x + 4 x2 y 3 ) 20 dy
R14 3
= 1 (16 + 8y 3 )dy
= (16y + 2y 4 ) 41
= 558

We evaluate the first integral above respect to x while treating y as a constant. Sim-
ilarly
" Z 2Z 4
(6x2 + 4xy 3 )dA = (6x2 + 4xy 3 )dydx = 558
R 0 1
!
Example 97 Evaluate the double integral R
(2x − 3y 2 )dA, where R = {(x, y) | 0 ≤
x ≤ 2, 1 ≤ y ≤ 2}.

By Brhane Aregawi Math Department, MU


55
4 Applied Math. II

Solution 48 Fubini’s Theorem gives


! R2R2
(2x − 3y 2 )dA = (2x − 3y 2 )dydx
R R02 1
= (2xy − y 3 ) 21 dx
R02
= 0 (2x − 7)dx
= x2 − 7x 20
= −10

Again applying Fubini’s Theorem, but this time integrating with respect to x first,
we have ! R2R2
R
(2x − 3y 2 )dA = 1 0 (2x − 3y 2 )dxdy
R2
= 1 (x2 − 3xy 2 ) 20 dy
R2
= 1 (4 − 6y 2 )dy
= 4y − 2y 3 21
= −10
R1R1 x−y R1R1 x−y
1
Example 98 Show that 0 0 (x+y)3
dydx = 2 and 0 0 (x+y)3
dxdy = − 12 ,

R1R1 x−y
Solution 49 First let’s evaluate 0 0 (x+y)3
dydx = 12 ,
R1R1 x−y R 1 R 1 2x−(x+y)
0 0 (x+y)3
dydx = (x+y)3
dydx
R01 R01 x+y d
2x
= 0 0 (x+y) 3 − x+y 3 ydx
R1R1
2x 1 d
= 0 0 (x+y) 3 − x+y 2 ydx
R1 
−x 1 1
= 0 (x+y) 2 − x+y 0 dx
R1
1
= 0 (x+1) dx
1 1
= − (x+1) 0
= 12
R1R1 x−y
and for 0 0 (x+y)3
dxdy = − 12 , we have
R1R1 x−y R1R1 y−x
0 0 (x+y)3
dxdy =− (x+y)3
dxdy
R01 R01 2y−(x+y)
= − 0 0 (x+y)3 dxdy
R 1 R 1  2y x+y

= − 0 0 (x+y)3 − (x+y)3 dxdy
R 1 R 1  2y 
1
= − 0 0 (x+y)3 − (x+y) 2 dxdy
R 1  −y 
1 1
= − 0 (x+y)2 − x+y 0 dxdy
R1
1
= − 0 y+1 dy
1 1
= y+1 0
= − 21

This shows that interchanging the order of integration may not always produce
equal results. A sufficient condition under which the order may be interchanged is

By Brhane Aregawi Math Department, MU


56
4 Applied Math. II

! x−y
that the double integral over the corresponding region exists. In this case R (x+y)3 dydx,
where R is the region 0 ≤ x ≤ 1, 0 ≤ y ≤ 1 fails to exist because of the discontinuity
of the integrand at (0,0). The integral is actually an improper double integral

4.1.3 Properties of double integrals


Let c be a constant, f and g be continuous functions on a region R on which
f (x, y) attains a minimum value m and a maximum value M. Let A(R) denote
area of the region R, if all the indicated integrals exist, then
! !
1. R cf (x, y)dA = c R f (x, y)dA.
! ! !
2. R (f (x, y) + g(x, y))dA = R f (x, y)dA + R g(x, y)dA.
!
3. m.A(R) ≤ R f (x, y)dA ≤ M.A(R).
! ! !
4. D f (x, y)dA = D f (x, y)dA + D f (x, y)dA
1 2

Figure 4.3: Distinict regions D1 and D2

! ! ! !
5. R
f (x, y)dA = R f (x, y)dA+ R f (x, y)dA+· · ·+ R , where R = R1 +R2 +
1 2 n
· · ·+Rn and the Ri ’s are mutually disjoint regions for each i ∈ {1, 2, 3, · · · , n}.
6. If f (x, y) ≥ g(x, y) for all (x, y) in D, then
" "
f (x, y)dA ≥ g(x, y)dA
D D

4.1.4 Double Integral over vertically and horizontally simple


regions
Definition 28 A plane region D is said to be vertically simple if there are two
continuous functions g1 and g2 on an interval [a, b] such that g1 (x) ≤ g2 (x) for
x ∈ [a, b] and such that D is the region between the graphs of g1 and g2 on [a, b]. In
this case we say that D is vertically simple region between the graphs of g1 and g2
on [a, b].

Definition 29 A plane region D is horizontally simple if there are two continu-


ous functions h1 and h2 on an interval [c, d] such that h1 (y) ≤ h2 (y) for y ∈ [c, d]
and such that D is the region between the graphs of h1 and h2 on an interval [c, d].
In this case we say that D is the horizontally simple region between the graphs of h1
and h2 on an interval [c, d].

By Brhane Aregawi Math Department, MU


57
4 Applied Math. II

Figure 4.4: Region D formed by g1 (x) and g2 (x)

Figure 4.5: Region D formed by h1 (x) and h2 (x)

By Brhane Aregawi Math Department, MU


58
4 Applied Math. II

Figure 4.6: Region D formed by y = 2x2 and y = x2 + 1

Definition 30 A plane region D is simple if it is both vertically simple and hori-


zontally simple.
Type I: Double Integral over Vertical Simple Region

Theorem 35 If f is continuous on type I region D such that D = {(x, y) | a ≤ b} and


{g1 (x) ≤ y ≤ g2 (x)}
" Z b Z g2 (x)
f (x, y)dA = f (x, y)dydx
D a g1 (x)

Type II: Double Integral over Horizontal Simple Region

Theorem 36 If f is continuous on type I region D such that D = {(x, y) | c ≤ d} and


{h1 (x) ≤ x ≤ h2 (x)}
" Z d Z h2 (x)
f (x, y)dA = f (x, y)dxdy
D c h1 (x)
!
Example 99 Evaluate D (x+2y)dA, where D is the region bounded by the parabo-
las y = 2x2 and y = x2 + 1.

Solution 50 The parabolas intersect when 2x2 = 1 + x2 ; that is, x2 = 1, so x = ±1.


We note that the region D, sketched in figure, is a type I region but not type II region
and we can write

D = {(x, y)| − 1 ≤ x ≤ 1, 2x2 ≤ y ≤ x2 + 1}

Since the lower boundary is y = 2x2 and the upper boundary is y = x2 + 1, then
! R 1 R x2 +1
D
(x + 2y)dA = 2x2
(x + 2y)dydx
R−1
1 2
x +1
= (xy + y 2 ) 2x 2 dx
R−1
1
= −1 (−3x − x + 2x2 + x + 1)dx
4 3
 5 4 3 2 
= −3 x5 − x4 + 2 x3 + x2 + x 1−1
32
= 15

By Brhane Aregawi Math Department, MU


59
4 Applied Math. II

Figure 4.7: Region formed by y = 2x and y = x2

Example 100 Find the volume of the solid that lies below the paraboloid z = x2 +y 2
above the region D in the xy-plane bounded by the line y = 2x and the parabolic
y = x2 .

Solution 51 From the figure below, we see that D is a type I region and

D = {(x, y)|0 ≤ x ≤ 2, x2 ≤ y ≤ 2x}

Therefore, the volume under z = x2 + y 2 and above D is


!
V = D (x2 + y 2 )dA
R 2 R 2x
= 0 x2 (x2 + y 2 )dydx
R2 
y 3 2x
2
= 0 x y + 3 x2 dx
R2 6 3
= 0 − x3 − x4 + 14 x3 dx
 7 5 4
x
= − 21 − x5 + 14 x4
= 216
35

Or D can also written as a type II region


1 √
D = {(x, y)|0 ≤ y ≤ 4, y ≤ x ≤ y}
2
Therefore another expressions for V is
!
V = D (x2 + y 2 )dA
R 4 R √y
= 0 1 (x2 + y 2 )dxdy
y
R 4  23  √y
x
= 0 3 + xy 2 1 dy
2y
R 4 y 32
!
5 y3 y3
= 0 − 3 + y − 24 − 2 dy
2

2 32 7
 
= 15 y + 72 y 2 − 13
96 y 4 4
0
= 216
35
R1R1 3
Example 101 Evaluate 0
y yex dxdy.
2

By Brhane Aregawi Math Department, MU


60
4 Applied Math. II


Figure 4.8: Region formed by x = 12 y and x = y

Solution 52 No one can not able to integrate with first respect to x, as indicated,
3
because ex is known to have no elementary anti-derivative. So we try to evaluate
the integral by first reversing the order of integration. To do so,
R1R1 R 1 R 2x
x3 =
3
yex dydx
y ye dxdy
0 2 0 0 
R 1 R 2x 3
= 0 0 yex dy dx
R 1 y2 3
= 0 2 ex 2x 0 dx
R1 3
2
= 0 2x e dx x
3
= 23 ex 10
= 23 (e − 1)

4.1.5 Change of Variables in Double Integral


In evaluating a multiple integral over a region R, it is often convenient to use
the coordinates other than rectangular, such as the polar coordinates, cylin-
drical coordinates, spherical coordinates, so on. If we let (u,v) to be curvi-
linear coordinates of points in a plane, there will be a set of transformation
x = f (u, v), y = g(u, v) mapping points (x, y) of the xy-plane into points (u,v) of
the uv-plane. In such case the region R of the xy-plane is mapped in to a region
R’ of the uv-plane. We then have
" "
∂(x, y)
F(x, y)dxdy = G(u, v) dudv
R R0 ∂(u, v)
∂x ∂x
∂(x,y) ∂u
where G(u, v) = F(f (u, v), g(u, v)) and ∂(u,v) = ∂y ∂v

∂y is the Jacobian of x and


∂u ∂v
y with respect to u and v. The result of the above corresponds to change of
variables for double integrals.

Definition 31 The Jacobian of the transformation T given by x = g(u, v) and y =


h(u, v) is
∂x ∂x
∂(x, y) = ∂u ∂v = ∂x ∂y − ∂x ∂y
∂(u, v) ∂y ∂y ∂u ∂v ∂v ∂u
∂u ∂v

By Brhane Aregawi Math Department, MU


61
4 Applied Math. II

!  x−y 4
Example 102 Compute D x+y dA, where D is the triangle region bounded by
the line x + y = 1 and the coordinate axes.

Solution 53 Let
u = x − y and v = x + y
V x = 12 (u + v) and y = 12 (u − v)
Now transform each boundary on D to the variables u and v; that is,
• On the y-axis, x = 0 V= −u.
• On the x-axis, y = 0 V v = u.

• On x + y = 1 V v = 1
The Jacobian of the change of variables on region D = {(u, v)|0 ≤ v ≤ 1, −v ≤ u ≤ v}
is
∂(x, y) xu xv 12 1

2 =
1
= =
∂(u, v) yu yv − 21 1
2 2
Therefore,
!  x−y 4 !  4
u 1
dA = dudv
D x+y R 1 Rv v 2
D
1 4 −4
= 2 0 −v u v dudv
1
= 10

∂(x,y)
Example 103 If u = x2 − y 2 and v = 2xy, find ∂(u,v) in terms of u and v.

∂(u,v) u uy
Solution 54 From the identity ∂(x,y)
= x = 4(x2 + y 2 ), we have
vx vy

(x2 + y 2 )2 = (x2 − y 2 )2 + (2xy)2



we have (x2 + y 2 )2 = u 2 + v 2 and x2 + y 2 = u 2 + v 2
Then
∂(x, y) 1 4 1
= = = √
∂(u, v) ∂(u,v) 4(x2 + y 2 ) 4 u 2 + v 2
∂(x,y)

This is because
∂(x, y) ∂(u, v)
· =1
∂(u, v) ∂(x, y)
∂(x,y)
provided that ∂(u,v)
, 0.

By Brhane Aregawi Math Department, MU


62
4 Applied Math. II

Figure 4.9: Polar Coordinates

Figure 4.10: R = {(x, y)|y ≥ 0, 1 ≤ x2 + y 2 ≤ 4}

4.1.6 Double Integral in Polar Coordinate


Let z = f (x, y) is continuous and is defined in the region R, then
" "
f (x, y)dA = f (rcosθ, rsinθ)rdrdθ
R R
Thus, we formally transform in to polar coordinates a double integral over a
polar rectangle of the form a ≤ r ≤ b, α ≤ θ ≤ β by substituting x = rcosθ, y =
rsinθ, dA = rdrdθ and inserting the appropriate limits of integration on r and
θ.
!
Example 104 Evaluate R (3x + 4y 2 )dA, where R is the region in the upper half-
plane bounded by the circles x2 + y 2 = 1 and x2 + y 2 = 4.

Solution 55 The region R can be described as


R = {(x, y)|y ≥ 0, 1 ≤ x2 + y 2 ≤ 4}
It is the half-ring shown in figure, and in polar coordinates it is given by 1 ≤ r ≤
, 0 ≤ θ ≤ π. Therefore,
! RπR2
R
(3x + 4y 2 )dA = 0 1 (3r cos θ + 4r 2 sin2 θ)rdrdθ
RπR2
= 0 1 (3r 2 cos θ + 4r 3 sin2 θ)dθdr

= 0 (r 3 cos θ + r 4 sin2 θ) 21 dθ

= 0 (7 cos θ + 15 sin2 θ)dθ

= 0 (7 cos θ + 152 (1 − cos 2θ)θ)dθ
= 7 sin θ 15
2 θ − 15
4 sin 2θ 0
π
15
= 2π

By Brhane Aregawi Math Department, MU


63
4 Applied Math. II

! p
Example 105 Evaluate R
x2 + y 2 dA, where R = {(x; y) | 1 ≤ x2 + y 2 ≤ 9, y ≥ 0.

Solution 56 Since R = {(x, y)|1 ≤ x2 +y 2 ≤ 9, y ≥ 0}, then R = {(r, θ)|1 ≤ r ≤ 3, 0 ≤


θ ≤ π}.
! p RπR3
R
x2 + y 2 dA = 0 1 rrdrdθ
RπR3
= 0 1 r 2 drdθ
RπR3 3
= 0 1 r3 31 dθ

= 0 263 θ
= 26 θ
3 0
θ
26
= 3θ

Example 106 Find the volume of the solid that lies under the paraboloid z = x2 +y 2 ,
above the xy-plane, and inside the cylinder x2 + y 2 = 2x.

Figure 4.11: Volume formed by z = x2 + y 2 and x2 + y 2 = 2x

Solution 57 The solid lies above the disk D whose boundary circle has equation
x2 + y 2 = 2x or, after completing the square,

(x − 1)2 + y 2 = 1

In polar coordinates we havex2 + y 2 = r 2 and x = r cos θ, so the boundary circle


becomes r 2 = 2r cos θ, or r = 2 cos θ. Thus the disk D is given by
π π
D = {(r, θ)| − ≤ θ , 0 ≤ r ≤ 2 cos θ}
2 2

By Brhane Aregawi Math Department, MU


64
4 Applied Math. II

Figure 4.12: Region D

and we have !
V = D
(x2 + y2)dA
Rπ 2 cos θ
R
= 2
− π2 0
r 2 rdrdθ
Rπ 4
2 r 2 cos θ
= − π2 4 0

π
R
=4 2 4
cos θdθ
− π2
π
R
=8 2 4
cos θdθ
R0π  2
2 1+cos 2θ
=8 2 dθ
R0π
=2 [1 + 2 cos 2θ + 21 (1 + cos 4θ)]dθ
2
h0 iπ
= 2 32 θ + sin 2θ + 18 sin 4θ 02
  
= 2 23 π2
= 3π
2

4.1.7 Application of Double Integral: Area, volume, Center of


Mass of Plane Region, Surface Area
Double integrals have different applications in solving practical problems. Some
of its applications are used to find area of a plane figure, volume of certain
solids and surface area of three dimensional surfaces, so on.
Area and volume by double integration

Definition 32 Suppose that a function f is continuous and non negative on the


bounded plane region R. Then the volume V of the solid that lies below the surface
z = f (x, y) and above the region R is defined to be
"
V = f (x, y)dA
R

provided that this integral exists.

Example 107 Let D be the solid region bounded above by the paraboloid z = 2 −
x2 − y 2 and below by the xy-plane. Find the volume V of D.

By Brhane Aregawi Math Department, MU


65
4 Applied Math. II

Figure 4.13: z = 2 − x2 − y 2

Solution 58 The region R of the equation is the disk x2 + y 2 ≤ 2. By using polar


coordinates !
V = R (2 − x2 − y 2 )dA
R 2π R √2
= 0 0 (2 − r 2 )rdrdθ
R 2π R √2
= 0 0 (2r − r 3 )drdθ
R 2π  √
3
= 0 r 2 − r3 0 2 drdθ
R 2π
= 0 dθ
= 2π

Definition 33 Suppose f (x, y) = 1 as a special case in the above equation. We


define the area A of a plan region R by
"
A= 1dA
R

where R is the region between the graphs of two continuous functions.

Example 108 Calculate area of the region R bounded by the cardioid r = 1 − cosθ.

Figure 4.14: r = 1 − cosθ

By Brhane Aregawi Math Department, MU


66
4 Applied Math. II

Solution 59 "
A = dA
R 2πRR 1−cos θ 
= 0 0
rdrd θ
R 2π  2 
r 1−cos θ
= 2 0 dθ
R02π
1+2 cos θ+cos2 θ
= 0 2 dθ
= 32 π
Surface Area of a Parametrized Surface
If a smooth parametrized surface S is given by the equation
r(u, v) = x(u, v)i + y(u, v)j + z(u, v)k for (u, v) ∈ D
and S is covered just once as (u, v) ranges throughout the parameter domain D,
then the surface area of S is
"
kru xrv kdA
D
∂x ∂y ∂z ∂x ∂y ∂z
where ru = ∂u
i + ∂u j + ∂u k and ru = ∂v
i + ∂v j + ∂v k.

Example 109 Find the surface area of a sphere of radius a, where the surface is
parametrized by
x = asinφcosθ, y = asinφsinθ, z = acosφ
where the parametric domain is
D = {(φ, θ) | 0 ≤ φ ≤ π, 0 ≤ θ ≤ 2π

Solution 60 We first compute the cross product of the tangent vectors

i j k

∂x ∂y ∂z

rφ × rθ = ∂φ ∂φ ∂φ
∂x ∂y ∂z
∂θ ∂θ ∂θ
= a2 sin2 φ cos θi +2 sin2 φ sin θj +2 sin2 φ cos φk
Thus
q
krφ × rθ k = a4 sin4 φ cosθ +a4 sin4 φ sin2 θ + a4 sin2 φ cos2 θ
q
= a4 sin4 φ + a4 sin2 φ cos2 φ
q
= a2 sin2 φ
= a2 sin φ
Since 0 ≤ φ ≤ π. Therefore, by the above definition, the area of the sphere is
!
S = D krφ × rθ kdA
R 2π R π
= 0 0 a2 sin φdφdθ
R 2π Rπ
= a2 0 dθ 0 sin φdφ
= a2 (2π)(2)
= 4πa2

By Brhane Aregawi Math Department, MU


67
4 Applied Math. II

Figure 4.15: z = 2 − x2 − y 2

Surface Area of The Graph of a Function

Definition 34 Let R be a vertically or horizontally simple region, and let f has


continuous partial derivatives on R. If S is the graph of f on R, then the surface
area A of S is defined by
" q
(fx (x, y))2 + (fy (x, y))2 + 1dA
R
.

Example 110 Find the surface area S of the portion of the paraboloid z = 2−x2 −y 2
that lies above the xy-plane.

Solution 61 The given surface lies over the region R in the xy-plane bounded by
the circle x2 + y 2 = 2. If f (x, y) = 2 − x2 − y 2 , then fx (x, y) = −2x and fy (x, y) = −2y.
So,
! q
S = R (fx (x, y))2 + (fy (x, y))2 + 1dA
! p
= R (−2x)2 + (−2y)2 + 1dA
! p
= R 4x2 + 4y 2 + 1dA
R 2π R √2 √
= 0 0 4r 2 + 1rdrdθ, let u = 4r 2 + 1 V du = 8rdr
R9√
= 2π8 R 1 udu
π 9√
= 4 1 udu
3
= π6 u 2 91
= 13
3 π
p
Example 111 Find the surface area S of the frustum of the cone z = a x2 + y 2 with
minimum and maximum radii r1 and r2 , respectively ans assume a > 0.

Solution 62 The given surface lies over the region R in the xy plane bounded by
the annual
r12 ≤ x2 + y 2 ≤ r22

By Brhane Aregawi Math Department, MU


68
4 Applied Math. II

p
If f (x, y) = a x2 + y 2 , then
ay
fx (x, y) = √ ax and fy (x, y) = √ 2
x +y 2
2 x +y 2
! q
S = (f (x, y))2 + (fy (x, y))2 + 1dA
Rs x
!
!2 !
ax ay
= R √ + √ + 1dA
x2 +y 2 x2 +y 2
!
r
2 x2 a2 y 2
= R xa2 +y 2 + x2 +y 2 + 1dA
! √
= R a2 + 1dA
√ !
= a2 + 1dA R dA

= π a2 + 1(r22 − r12 )
since the area of the annuales R is π(r22 − r12 ).
Calculation of Mass
For a plane lamina, if the surface density at the point P (x, y) is ρ = f (x, y), then
the elementary mass at P is equal to ρdxdy. Therefore, the total mass of the
lamina is "
m= ρdxdy

with integrals embracing the whole area of the lamina. In polar coordinates,
taking ρ = φ(r, θ) at the point P (r, θ), the total mass of the lamina is
"
m= ρrdrdθ.

Center of Gravity
To find the center of gravity (x, y) of a plane lamina, take the element of mass
ρdxdy at the point P (x, y), then
! !
1 1
x= m xρdxdy and y = m yρdxdy

Example 112 Find the mass and center of mass of a triangular lamina with vertex
(0, 0), (1, 0) and (0, 2) if the density function is ρ(x, y) = 1 + 3x + y.

Solution 63 The triangle is shown in the figure below. Note that the equation of
the upper boundary is y = 2 − 2x. The mass of the lamina is
!
m = D ρ(x, y)dA
R 1 R 2−2x
= 0 0 (1 + 3x + y)dydx
R1 
= 0 y + 3xy + 12 y 2 02−2x dx
R1
= 0 (1 − x2 )dx
 3
= 4 x − x3 10
= 83

By Brhane Aregawi Math Department, MU


69
4 Applied Math. II

Figure 4.16: Triangular lamina

Then !
1
x = m R DR xρ(x, y)da
3 1 2−2x
=8 0 0 (x + 3X 2 + xy)dydx
R1 y2

= 38 0 xy + 3X 2 y + x 2 2−2x 0 dydx
3 1
R
= 8 0 (x − x3 )dx
 2 4
= 38 x2 − x4 10
= 83
!
1
y =m yρ(x, y)da
R DR
3 1 2−2x
=8 0 0 (y + 3Xy + y 2 )dydx
R 1  y2 y2 y3

= 38 0 2 + 3X 2 + 3 2−2x 0 dydx
R1
= 14 0 (7 − 9x − 3x2 + 5x3 )dx
 2 4
= 41 7x − 9 x2 − x3 + 5 x4 10
= 1116

Example 113 The density at any point on a semicircular lamina is proportional to


the distance from the center of the circle. Find the center of mass of the lamina.
Solution: Let’s place the lemina as the upper half of the circle x2 + y 2 = p a2 . Then
the distance from a point (x, y) to the center of the circle (the origin) is x2 + y 2 .
Therefore the density function is
q
ρ(x, y) = K x2 + y 2

where K is some constant. Both the density functionp and the shape of the lamina
suggest that we convert to polar coordinates. Then x2 + y 2 = r and the region D is
given by 0 ≤ r ≤ a, 0 ≤ θ ≤ π. Thus the mass of the lamina is
!
m = D ρ(x, y)dxdy
! p
= D K x2 + y 2 dxdy
RπRa
= 0 0 (Kr)rdrdθ
Rπ Ra
= K 0 dθ 0 r 2 dr
3
= Kπ r3 a
0
3
= Kπa
3

By Brhane Aregawi Math Department, MU


70
4 Applied Math. II

Figure 4.17: Semicircular lamina

Both the lamina and the density function are symmetric with respect to the y-axis,
so the center of mass lie on the y-axis; that is, x=0. The y-coordinate is given by
!
1
y =m yρ(x, y)dxdy
DR π R a
3
= Kπa3 0 0 r sin θ(Kr)rdrdθ
Rπ Ra
3 3
= Kπa 3 0 sin θdθ 0 r dr
π r4 a
 
3
= Kπa3 (− cos θ) 0 4 0
3 2a4
= Kπa3 4
3a
= 2π

Moment of Inertia
Let a particle of mass m of a body be at a distance r from a given line, then
mr 2 is called the moment of inertia of the particle about the given line and
P the
sum of similar expressions taken for all the particles of the body; that is, mr 2
is called the moment of inertial of the body about the given line. If m is the
total mass of the body and we write its moment of inertia is equal to mk 2 , then
k is called the radius of gyration of the body about the axis.
Moment of inertial of a plane lamina
Consider an elementary mass ρdxdy at the point P (x, y) of a plane area A so
that it’s Moment of inertial about x-axis is equal to ρy 2 dxdy. Therefore, Mo-
ment of inertial of the lamina about x-axis, Ix is given by
"
Ix = ρy 2 dxdy

Similarly, Moment of inertial of the lamina about the y-axis; that is, Iy is given
by "
Iy = ρx2 dxdy

Also Moment of inertial of the lamina about an axis perpendicular to the xy-
plane is "
Iz = ρ(x2 + y 2 )dxdy

Example 114 Find the moments of inertia Ix , Iy and I0 of a homogeneous disk D


with density ρ(x, y) = ρ, center the origin, and radius a.

By Brhane Aregawi Math Department, MU


71
4 Applied Math. II

Solution 64 The boundary of D is the circle x2 + y 2 = a2 and in polar coordinates


D is described by 0 ≤ θ ≤ 2π, 0 ≤ r ≤ a. Let’s compute I0 first
!
I0 = D ρ(x2 + y 2 )dxdy
R 2π R a
= ρ 0 0 r 2 rdrdθ
R 2π Ra
= ρ 0 dθ 0 r 3 dr
 4
= 2πρ r4 a0
πρa4
= 2

Instead of computing Ix and Iy directly, we use the facts that Ix + Iy = I0 and Ix = Iy


I0 πρa4
from the symmetry of the problem. Thus Ix = Iy = 2 = 4 .

4.2 Triple Integral


4.2.1 Notation of Triple Integral
Definition 35 The triple integral of a function f (x, y, z) defined in a closed three
dimensional region D and is given by
" l X
X m X
n
f (x, y, z)dv = l,m,n→∞ f (xi∗ , yj∗ , zk∗ )∆V
D i=1 j=1 k=1

is called the triple integral of f (x, y, z) over D. The limit exist if f is continu-
ous(sectionally continuous) in D.

Theorem 37 (Fubini’s Theorem for triple integral) Suppose that f (x, y, z) is con-
tinuous on the rectangular box B = {(x, y, z) | a ≤ x ≤ b, c ≤ y ≤ d, e ≤ z ≤ f , then
$ Z bZ dZ f
f (x, y, z)dv = f (x, y, z)dzdydx.
B a c e

Example 115 Let R be the rectangular region in the xy-plane bounded by the line
x = 61 , x = 1, y = 0 and y = π,!and let D be the parallelepipeds between the graph of
z=0 and z=2 on R. Evaluate D zx sin(xy)dv.

Solution 65
! R R
π 2
 
D
zx sin(xy)dv = int 11 0 0 (zx sin(xy))dz dy dx
R 1 6R π   
= 1 0 21 z2 x sin(xy) 20 dy dx
R 61 R π 
= 1 0 (2x sin(xy))dy dx
R 61
= 1 (2 cos(xy)) π 0 dx
R 61
= 1 (2 − 2 cos(πx))dx
6 
= 2x − π2 sin(πx) 11
 6
= 2 − 13 − π2 sin π6
 
= 2 − 13 − π1
= 53 + π1

By Brhane Aregawi Math Department, MU


72
4 Applied Math. II

Theorem 38 (Evaluation of Triple Integral in iterated integral) The triple in-


tegral over R given by the above as an iterated integral of the form
Z b Z g2 (x) Z u2 (x,y) Z b Z g2 (x) Z u2 (x,y)  
   
f (x, y, z)dzdydx = 


 f (x, y, z)dz dy  dx
a g1 (x) u1 (x,y) a g1 (x) u1 (x,y)

Theorem 39 (Evaluation of a Triple Integral over any z-simple Region) If f (x, y, z)


is continuous on the z-simple region D defined by (x, y) ∈ Rxy , g1 (x, y) ≤ z ≤ g2 (x, y),
where Rxy is a normal region in the xy-plane, then
$ " Z g2 (x,y)
f (x, y, z)dv = f (x, y, z)dz
D xy g1 (x,y)

Theorem 40 [Evaluation of a Triple Integral over any y-simple Region]


If f (x, y, z) is continuous on the y-simple region D defined by (x, z) ∈ Rxz , h1 (x, z) ≤
y ≤ h2 (x, z), where Rxz is a normal region in the xz-plane, then
$ " Z h2 (x,z)
f (x, y, z)dv = f (x, y, z)dy
D xz h1 (x,z)

Theorem 41 [Evaluation of a Triple Integral over any x-simple Region] If


f (x, y, z) is continuous on the x-simple region D defined by (y, z) ∈ Ryz , h1 (y, z) ≤
y ≤ h2 (y, z), where Ryz is a normal region in the yz-plane, then
$ " Z k2 (y,z)
f (x, y, z)dv = f (x, y, z)dx
D yz k1 (y,z)

Evaluation of triple integrals as they may some times be difficult or even im-
possible. In this case changing one coordinate system to another may facil-
itate the process. If (u,v,w) are curvilinear coordinates in three dimensions,

Figure 4.18: Region R

there will be a set of transformation equations, x = f (u, v, w), y = g(u, v, w) and


z = h(u, v, w) and we can write
$ $
∂(x, y, z)
f (x, y, z)dv = G(u, v, w)dv
dudvdw
D D0 ∂(u, v, w)

By Brhane Aregawi Math Department, MU


73
4 Applied Math. II

where G(u, v, w) = F(f (u, v, w), g(u, v, w), z = h(u, v, w)) and
∂x ∂x ∂x
∂u ∂v ∂w
∂(x, y, z) = ∂y ∂y ∂y
∂(u, v, w) ∂u ∂v ∂w
∂z ∂z ∂z
∂u ∂v ∂w

is the Jacobian of x,y and z with respect to u,v and w. The result of the triple
integral corresponds to change of variables for triple integrals.

4.2.2 Triple Integral in Cylindrical Coordinate


Let D be a solid region between the graphs of f1 and f2 on R, where R is the
plane region between the polar graphs of h1 and h2 on [α, β], with 0 ≤ β−α ≤ 2π
and 0 ≤ h1 (θ) ≤ h2 (θ) for α ≤ θ ≤ β. If f is continuous on D, then
$ Z β Z h2 (θ) Z F2 (rcosθ,rsinθ)
f (x, y, z)dv = f (rcosθ, rsinθ, z)rdzdrdθ
D α h1 (θ) F1 (rcosθ,rsinθ)

Figure 4.19: Cylinderical coordinate

R 2 R √4−x2 R 2
Example 116 Evaluate √ √ (x2 + y 2 )dzdydx.
−2 − 4−x2 x2 +y 2

Solution 66 This iterated integral is a triple integral over the region


√ √ q
E = {(x, y, y) | −2 ≤ x ≤ 2, − 4 − x2 ≤ y ≤ 4 − x2 , x2 + y 2 ≤ z ≤ 2}

and the projection of 2 2


p E on to the xy-plane is the disk x + y ≤ 4. The lower surface
of E is the cone z = 2 2
+ y and its upper surface is the plane z = 2. This region has a
much simpler description in cylindrical coordinates E = (r, θ, z)|0 ≤ θ ≤ 2π, 0 ≤ r ≤ 2, r ≤ z ≤ 2
Therefore, we have
R 2 R √4−x2 R 2 !
√ √ (x2 + y 2 )dzdydx = (x2 + y 2 )dA
−2 − 4−x2 x2 +y 2 E
R 2π R 2 R 2
= 0 0 r r 2 rdrdθ
R 2π R2R2
= 0 dθ 0 r r 3 (2 − r)dr
 
= 2π 12 r 4 − 51 r 5 20
= 16
5 π

By Brhane Aregawi Math Department, MU


74
4 Applied Math. II

√ √ p
Figure 4.20: E = {(x, y, y) | −2 ≤ x ≤ 2, − 4 − x2 ≤ y ≤ 4 − x2 , x2 + y 2 ≤ z ≤ 2}

4.2.3 Triple Integral in Spherical Coordinate


Theorem 42 [Triple Integrals in Spherical Coordinates]
If f (x;y; z) is continuous on a solid region D, then
$ $
f (x, y, z)dv = f (rsinφcosθ, rsinφsinθ, rcosφ)r 2 sinφdrdφdθ
D D0

Then the point P is said to have spherical coordinates r, θ and φ, and write P =
(ρ, θ, φ) as well as P = (x, y, z), where 0 ≤ r ≤ ∞, 0 ≤ θ ≤ 2π and 0 ≤ φ ≤ π. So,
x = rcosθ = rsinφcosθ, y = rsinθ = rsinφsinθ and z = rcosφ.

Figure 4.21: Spherical coordinate

The element in spherical coordinates dV = r2 sinφdrdφdθ.


# 2 +y 2 +z2 )3/2
Example 117 Evaluate B
e(x dV , where B is the ball B = {(x, y, z) | x2 +
y 2 + z2 ≤ 1}.

Solution 67 Since the boundary of B is a sphere, we use spherical coordinates

B = {(ρ, θ, φ)|0 ≤ ρ ≤ 1, 0 ≤ θ ≤ 2π, 0 ≤ φ ≤ π}

By Brhane Aregawi Math Department, MU


75
4 Applied Math. II

Figure 4.22: Element dV

In addition, spherical coordinates are appropriate because x2 + y 2 + z2 = ρ2 . This


gives
# 2 2 2 3/2 R π R 2π R 1 2 3/2
B
e(x +y +z ) dV = 0 0 0 e(ρ ) ρ2 sin φdρdθdφ
Rπ R 2π R1 3
= 0 sin φdφ 0 dθ 0 ρ2 eρ dρ
3
h i
1 ρ 1
= [− cos φ] π0 (2π) 3 e 0
4
= 3 π(e − 1)

Example 118 Use spherical coordinates to find the volume of the solid that lies

above the cone z = x2 + y2 and below the sphere x2 + y2 + z2 = z.

Figure 4.23: Volume of the solid

Solution 68 Notice that the sphere passes through the origin and has center (0, 0, 12 ).
We write the equation of the sphere in spherical coordinates as
ρ2 = ρcosφ or ρcosφ

The equation of the cone can be written as


q
ρcosφ = ρ2 sin2 φcos2 θ + ρ2 sin2 φsin2 θ = ρsinφ

By Brhane Aregawi Math Department, MU


76
4 Applied Math. II

This gives sinφ = cosφ, or φ = π4 . Therefore the description of the solid E in spheri-
cal coordinates is
π
E = {(ρ, θ, φ) | 0 ≤ 2π, 0 ≤ , 0 ≤ ρ ≤ cosφ}
4
E is swept out if we integrate first respect to ρ, then φ and then θ. Then volume of

Figure 4.24: Volume, where ρ, φ and θ varies respectively

E is
$ Z 2π Z π Z cosφ
4
V (E) = dV = ρ2 sinφdρdφdθ
E 0 0 0
π
=
8

4.2.4 Application of Triple Integral: Volume, Center of Mass


of Solid Region
Volume by Triple Integral
Suppose f (x, y, z) = 1 in the above equation. The volume of solid D is defined
to be $
dV
D

Example 119 Use triple integral to find the volume of the tetrahedron T bounded
by the planes x + 2y + z = 2, x = 2y, x = 0 and z = 0.

Solution 69 The tetrahedron T and its projection D on the xy-plane are shown in
the figure below. The lower boundary of T is the plane z = 0 and the upper boundary
is the plane x + 2y + z = 2, that is,z = 2 − x − 2y. Therefore we have

By Brhane Aregawi Math Department, MU


77
4 Applied Math. II

Figure 4.25: Tetrahedron

Figure 4.26: Region D formed by y = 1 − 2x and y = x


2

$
V (T ) = dv
T
Z 1Z 1− 2x Z 2−x−2y
= dzdydx
0 x 0
2
Z 1Z 1− 2x
= (2 − x − 2y)dydx
0 x
2
1
=
3
Calculation of Mass
For a solid, if the density at a point P (x, y, z) is ρ = f (x, y, z), then total mass of
the solid is $
m= ρdxdydz

with appropriate limits of integration.


Center of Gravity
To find the center of gravity (x, y, z) of a solid, take an element of mass ρdxdydz

By Brhane Aregawi Math Department, MU


78
4 Applied Math. II

enclosing the point P (x, y, z), then


$
1
x= xρdxdydz,
m
$
1
y= yρdxdydz and
m
$
1
z= zρdxdydz
m
Moment of Inertia
Let a particle of mass m of a body be at a distance r from a given line, then
mr 2 is called the moment of inertia of the particle about the given line and P the
sum of similar expressions taken for all the particles of the body; that is, mr 2
is called the moment of inertial of the body about the given line.
If m is the total mass of the body and we write its moment of inertia is equal
to mk 2 , then k is called the radius of gyration of the body about the axis.
Moment of Inertia of a solid
Consider elementary mass ρdxdydz enclosing a point p P (x, y, z) of a solid from
volume V. Distance of P from the x-axis is equal to x2 + y 2 . Therefore, Mo-
ment of Inertia of this element about the x-axis is equal to#ρdxdydz(y 2 + z2 ).
Thus, Moment of Inertia of this solid about the x-axis is Ix = ρ(y 2 +z2 )dxdydz.
#
Similarly Moment of Inertia about the y-axis is Iy = ρ(x2 + z2 )dxdydz and
#
Moment of Inertia about the z-axis is Iz = 2 2
ρ(x + y )dxdydz.

Example 120 Find the center of mass of a solid of constant density that is bounded
by the parabolic cylinder x = y 2 and the planes x = z, z = 0 and x = 1.

Solution 70 The solid E and its projection on to the xy-plane are shown in the
figure below. The lower and upper surfaces of E are planes z = 0 and z = x, so we
described E as a type I region

E = {(x, y, z) | −1 ≤ y ≤ 1, y 2 ≤ x ≤ 1, 0 ≤ z ≤ x}

Then, if the density is ρ(x, y, z) = ρ, the mass is


$ Z 1 Z 1Z x

m= ρdV = ρdzdxdy =
−1 y2 0 5

Because of the symmetry of E and r about the xy-plane, we can immediately say that
Iy = 0 and therefore y = 0. The other moments are
#
Ix = E xρdv
R1 R1 Rx
= −1 y 2 0 xρdzdxdy

= 7
#
Iz = zρdv
R 1 ER 1 R x
= −1 y 2 0
zρdzdxdy

= 7

By Brhane Aregawi Math Department, MU


79
4 Applied Math. II

Figure 4.27: E = {(x, y, z) | −1 ≤ y ≤ 1, y 2 ≤ x ≤ 1, 0 ≤ z ≤ x}

Therefore the center of mass is

Ix Iy Iz
! 
5 5

(x, y, z) = , , = , 0,
m m m 7 14

Example 121 Use a triple integral to find the volume of the solid enclosed between
the cylinder x2 + y 2 = 16 and the planes z = 0 and y + z = 4.

Solution 71 The solid D and its projection R on the xy-plane. The lower surface
of the solid is the plane z = 0, and the upper surface is the plane y + z = 4, or
equivalently, z = 4 − y. Thus
#
V = D dv
! R 4−y 
= R 0 dz dA
!
= R (4 − y)dA
R 2π R 4
= 0 0 (4 − r sin θ)rdrdθ, by using cylindrical coordinate
R 2π R 4 
= 0 (4r − r 2 sin θ)rdr dθ
0
R 2π  3 
= 0 2r 2 − r3 sin θ 40 dθ
R 2π  
= 0 32 − 64 3 sin θ dθ
 

= 32θ + 64 3 cos θ 0
= 64π

Example 122 Find the volume V of the region in the first octant bounded by the
planes z = 10 + x + y, y = 2 − x, y = x, z = 0 and x = 0.

Solution 72 The region R of the solid bounded in the xy-plane. The line y = 2 − x

By Brhane Aregawi Math Department, MU


80
4 Applied Math. II

and y = x intersects at (1,1). Hence, x ≤ y ≤ 2 − x as 0 ≤ x ≤ 1. Therefore,


#
V dv
R 1 R 2−x R 10+x+y  
= 0 x 0
dz dy dx
R 1 R 2−x 
= 0 x (10 + x + y)dy dx
R 1 R 2−x 
= 0 x (10 + x + y)dy dx
R1
= 0 (22 − 20x − 2x2 )dx
 3
= 2 11x − 5x2 − x3 10
= 34
3

Example 123 Let D be the solid region bounded by the circular paraboloids
# z=
3 − x2 − y 2 and z = −5 + x2 + y 2 for which x ≥ 0 and y ≥ 0. Evaluate ydv.

Solution 73 First find the intersection points of the two paraboloids.

3 − x2 − y 2 = −5 + x2 + y 2
V x2 + y 2 = 4

But if x2 +y 2 = 4, then z = 3−(x2 +y 2 ) = 3−4 = −1. We find that the corresponding


region R in the xy-plane is the horizontal simple region R in the first quadrant
thatplies inside the circle x2 + y 2 = 4, and hence between the graphs of x = 0 and
x = 4 − y 2 for 0 ≤ y ≤ 2. Since 3 − x2 − y 2 ≥ −5 + x2 + y 2 for (x,y) in R, we have
# R 2 R √4−y R 3−x2 −y 2
ydv = 0 0 −5+x2 +y 2
ydzdxdy
R 2 R √4−y 3−x2 −y 2
= 0 0
yz −5+x2 +y 2 dxdy
R 2 R √4−y
= y(8 − 2x2 − 2y 2 )dxdy
R02 0 3
= 0 y(8 − y 2 ) 4 − y − 12 y(4 − y 3 ) 2 dy
p

= 128
15

R3R1
Exercise 6 1. Evaluate the iterated integral 1 0
(1 + 4xy)dxdy.

Ans. 10
! xy 2
2. Evaluate the double integral R x2 +1
dA, R = {(x, y)|0 ≤ x ≤ 1, −3 ≤ y ≤ 3}.

Ans. 9 ln 2

3. Find the volume of the solid that lies under the plane 3x + 2y + z = 12 and
above the rectangle R = {(x, y)|0 ≤ x ≤ 1, −2 ≤ y ≤ 3}.

Ans. 47.5

x2 y2
4. Find the volume of the solid lying under the elliptic paraboloid 4 + 9 +z = 1
and above the rectangle R = [−1, 1] × [−2, 2].

By Brhane Aregawi Math Department, MU


81
4 Applied Math. II

166
Ans. 27
!
5. Evaluate the double integral D
x cos ydA, D is bounded by y = 0, y = x2 ,
x = 1.

Ans. 12 (1 − cos 1

6. Find the volume of the solid under the surface z = xy and above the triangle
with vertex (1,1), (4,1), and (1,2).

Ans. 6
R1R3 2
7. Evaluate the integral 0 3y
ex dxdy.

Ans. 16 (e9 − 1)
! 2 2
8. Evaluate the integral R e−x −y dA, where D is the region bounded by the
p
semi circle x = 4 − y 2 and y-axis by changing to polar coordinate.

π −4
Ans. 2 (1 − e )
p
9. Find the volume of of the solid above the cone z = x2 + y 2 and below the
sphere x2 + y 2 + z2 = 1.
  

Ans. 3 1 − √1
2

10. Use polar coordinate to combine the sum


√ √
Z 1 Z x Z 2Z x Z 2 Z 4−x2
√ xydydx + xydydx + √ xydydx
√1 x2 1 ) 2 0
2

into one double integral. Ten evaluate the double integral.

15
Ans. 16

11. Find the mass and center of mass of the lamina that occupies the region D and
has the density ρ(x, y) = x + y, where D is the triangular region with vertices
(0,0), (2,1), (0,3).
 
Ans. 6, 43 , 32
#
12. Evaluate the triple integral E 6xydV , where E lies under the plane z = 1+x+

y and above the region in the xy-plane bounded by the curves y = x, y = 0,
and x = 1.

65
Ans. 28

By Brhane Aregawi Math Department, MU


82
4 Applied Math. II

#
13. Evaluate the triple integral E xdV , where E bounded by the paraboloid x =
4y 2 + 4z2 and the plane x = 4.

16π
Ans. 3

14. Express the volume of the wedge in the first octant that is cut from the cylinder
y 2 + z2 = 1 by the planes y = x and x = 1 as a triple integral.

R x R x R √1−x2
Ans. 0 0 0 dzdydx

15. Find the mass and center of mass of the solid E with the density ρ(x, y, z) =
x2 + y 2 + z2 , where E is the cube given by 0 ≤ x 5 a, 0 ≤ y ≤ a, 0 ≤ z ≤ a.
 
7a 7a 7a
Ans. a5 , 12 , 12 , 12

# p
16. Evaluate E x2 + y 2 dV by using cylindrical coordinate, where E is the re-
gion that lies inside the cylinder x2 + y 2 = 16 and between the planes z = −5
and z = 4.

384π
Ans. 17
#
17. Evaluate E x2 dV by using cylindrical coordinate, where E is the solid that
lies within the cylinder x2 +y 2 = 1, above the planes z = 0, and below the cone
z2 = 4x2 + 4y 2 .


Ans. 5
#
18. Evaluate B (x2 + y 2 + z2 )dV by using spherical coordinate, where B is the
ball with center the origin and radius 5.


Ans. 5
!
19. Use the given transformation to evaluate the integral R x2 dA, where R is the
region bounded by the ellipse 9x2 + 4y 2 = 36; x = 2u, y = 3v.

Ans. 6π
!  y−x 
20. Evaluate R cos y+x dA by appropriate change of variables, where R is the
trapezoidal region with vertex (1,0), (2,0), (0,2), (0,1).

3
Ans. 2 sin 1

By Brhane Aregawi Math Department, MU


83

You might also like